APEA Predictor Exam 1014 Questions and Correct Answers | Exams Nursing | Docsity (2024)

  • Prepare for your exams

  • Get points

  • Guidelines and tips

  • Sell on Docsity
Log inSign up

APEA Predictor Exam 1014 Questions and Correct Answers | Exams Nursing | Docsity (2)

APEA Predictor Exam 1014 Questions and Correct Answers | Exams Nursing | Docsity (3)

Prepare for your exams

Study with the several resources on Docsity

APEA Predictor Exam 1014 Questions and Correct Answers | Exams Nursing | Docsity (4)

Earn points to download

Earn points by helping other students or get them with a premium plan

APEA Predictor Exam 1014 Questions and Correct Answers | Exams Nursing | Docsity (5)

Guidelines and tips

Sell on Docsity
Log inSign up

APEA Predictor Exam 1014 Questions and Correct Answers | Exams Nursing | Docsity (7)

Prepare for your exams

Study with the several resources on Docsity

Find documentsPrepare for your exams with the study notes shared by other students like you on DocsitySearch Store documentsThe best documents sold by students who completed their studies

Search through all study resources

Docsity AINEWSummarize your documents, ask them questions, convert them into quizzes and concept mapsExplore questionsClear up your doubts by reading the answers to questions asked by your fellow students

APEA Predictor Exam 1014 Questions and Correct Answers | Exams Nursing | Docsity (8)

Earn points to download

Earn points by helping other students or get them with a premium plan

Share documents20 PointsFor each uploaded documentAnswer questions5 PointsFor each given answer (max 1 per day)

All the ways to get free points

Get points immediatelyChoose a premium plan with all the points you need

Study Opportunities

Search for study opportunitiesNEWConnect with the world's best universities and choose your course of study

Community

Ask the communityAsk the community for help and clear up your study doubts University RankingsDiscover the best universities in your country according to Docsity users

Free resources

Our save-the-student-ebooks!Download our free guides on studying techniques, anxiety management strategies, and thesis advice from Docsity tutors

From our blog

Exams and Study

Go to the blog

A.T. Still University of Health Sciences (ATSU)Nursing

APEA Predictor Exam 1014 Questions and Correct AnswersAPEA Predictor Exam 1014 Questions and Correct AnswersAPEA Predictor Exam 1014 Questions and Correct Answers

Typology: Exams

2023/2024

1 / 104

APEA Predictor Exam 1014 Questions and Correct Answers | Exams Nursing | Docsity (12)

Related documents

APEA Predictor Exam 1014 Questions and Correct Answers | Exams Nursing | Docsity (13)

2024 APEA FNP PREDICTOR Exam Test Bank with 300+ Questions from Actual Past Exam

APEA Predictor Exam 1014 Questions and Correct Answers | Exams Nursing | Docsity (14)

APEA Predictor Exam 1014 Questions and Correct Answers | Exams Nursing | Docsity (15)

APEA 3P TEST BANK QUESTIONS ANDCORRECT ANSWERS

(1)

APEA Predictor Exam 1014 Questions and Correct Answers | Exams Nursing | Docsity (16)

APEA Predictor Exam 1014 Questions and Correct Answers>

APEA Predictor Exam 1014 Questions and Correct Answers | Exams Nursing | Docsity (17)

APEA Predictor Exam 1014 Questions and Correct Answers>

APEA Predictor Exam 1014 Questions and Correct Answers | Exams Nursing | Docsity (18)

APEA Predictor Exam 1014 Questions and Correct Answers>

APEA Predictor Exam 1014 Questions and Correct Answers | Exams Nursing | Docsity (19)

APEA Predictor Exam 1014 Questions and Correct Answers>

APEA Predictor Exam 1014 Questions and Correct Answers | Exams Nursing | Docsity (20)

APEA Predictor Exam 1014 Questions and Correct Answers>

APEA Predictor Exam 1014 Questions and Correct Answers | Exams Nursing | Docsity (21)

APEA Predictor Exam 1014 Questions and Correct Answers>

APEA Predictor Exam 1014 Questions and Correct Answers | Exams Nursing | Docsity (22)

APEA Predictor Exam 1014 Questions and Correct Answers

APEA Predictor Exam 1014 Questions and Correct Answers

APEA Predictor Exam 1014 Questions and Correct Answers | Exams Nursing | Docsity (24)

APEA Predictor test exam 1014Questions and Answers with complete solution(Graded A+)

APEA Predictor Exam 1014 Questions and Correct Answers | Exams Nursing | Docsity (25)

AORN PERIOP 101 Exam - APEA Predictor Exam with Correct Questions and Answers

APEA Predictor Exam 1014 Questions and Correct Answers | Exams Nursing | Docsity (26)

APEA Predictor Exam 2023 Questions and Answers(Verified Answers)

APEA Predictor Exam 1014 Questions and Correct Answers | Exams Nursing | Docsity (27)

NR 603 APEA PREDICTOR EXAM QUESTIONS AND ANSWERS WITH EXPLANATIONS

APEA Predictor Exam 1014 Questions and Correct Answers | Exams Nursing | Docsity (28)

APEA PREDICTOR EXAM QUESTIONS AND ANSWERS 100% VERIFIED RATED A++

APEA Predictor Exam 1014 Questions and Correct Answers | Exams Nursing | Docsity (29)

BUNDLE FOR 3 Defferent APEA Predictor, APEA 3P TEST BANK Exams Complete Questions And Veri

APEA Predictor Exam 1014 Questions and Correct Answers | Exams Nursing | Docsity (30)

BUNDLE FOR 3 Defferent APEA Predictor, APEA 3P TEST BANK Exams Complete Questions And Veri

APEA Predictor Exam 1014 Questions and Correct Answers | Exams Nursing | Docsity (31)

APEA Predictor Exam 1014 Questions and Correct Answers | Exams Nursing | Docsity (32)

APEA Predictor 1770 Complete Questions And Verified Answers

APEA Predictor Exam 1014 Questions and Correct Answers | Exams Nursing | Docsity (33)

APEA Predictor 1770 Complete Questions And Verified Answers

APEA Predictor Exam 1014 Questions and Correct Answers | Exams Nursing | Docsity (34)

APEA Predictor 1770 Complete Questions And Verified Answers

APEA Predictor Exam 1014 Questions and Correct Answers | Exams Nursing | Docsity (35)

APEA Predictor 1770 Complete Questions And Verified Answers

APEA Predictor Exam 1014 Questions and Correct Answers | Exams Nursing | Docsity (36)

APEA Predictor 1770 Complete Questions And Verified Answers

APEA Predictor Exam 1014 Questions and Correct Answers | Exams Nursing | Docsity (37)

APEA Predictor 1770 Complete Questions And Verified Answers

APEA Predictor Exam 1014 Questions and Correct Answers | Exams Nursing | Docsity (38)

APEA Predictor 1770 Complete Questions And Verified Answers

APEA Predictor Exam 1014 Questions and Correct Answers | Exams Nursing | Docsity (39)

APEA Predictor 1770 Complete Questions And Verified Answers

APEA Predictor Exam 1014 Questions and Correct Answers | Exams Nursing | Docsity (40)

APEA Predictor Exam 1014 Questions and Correct Answers | Exams Nursing | Docsity (41)

APEA Predictor Exam 1014 Questions and Correct Answers | Exams Nursing | Docsity (42)

Partial preview of the text

Download APEA Predictor Exam 1014 Questions and Correct Answers and more Exams Nursing in PDF only on Docsity! APEA Predictor Exam 1014 Questions and Correct Answers Cranial Nerve VIII - Correct answerAcoustic (Hearing and balance) Cranial nerve IX - Correct answerGlossopharyngeal- swallowing and rise of the palate, gag reflex Cranial Nerve XI - Correct answerSpinal Accessory (upper trapezius) Deep tendon reflex - Correct answerAnkle- S1 primarily Knee- Lumbar 2,3,4 Supinator(brachioradialis)-C5,6 BicepTriceps-C6,7 Stroke risk scale - Correct answerCHADS2 C-Congestive heart failure H-Hypertension A-Age >75 years D- Diabetes S-prior Stroke/TIA In the majority of children, the first permanent teeth start to erupt at the age of 6 years. Which of the following are the first permanent teeth to erupt in this time period? A. First Molars B. Second Molars C. Lower or upper incisors D. Canines - Correct answerA. First Molars All of the following are classified as activities of daily living (ADLs) except: A. ability to feed self B. Ability to manage bladder and bowel elimination C. Personal hygiene and grooming D. Grocery Shopping - Correct answerD. Grocery shopping You note the following result on a routine urinalysis of a 37-year old primigravida who is at 30 weeks gestation. Leukocyte=trace, nitrite=negative, protein=2+, blood=negative. Her weight has increased by 5 lbs during the past week. Which of the following is most likely? A.HELLP syndrome B. Pregnancy-induced hypertension (pre-eclampsia) C. Eclampsia of pregnancy D. Primary hypertension - Correct answerB. Pregnancy-induced hypertension (pre- eclampsia). Classic triad of symptoms of preeclampsia include hypertension, edema (weight gain), and proteinuria. The mother of a 12 month old infant reports to the nurse practitioner that her child had a high fever for several days, which spontaneously resolved. After the fever resolved, the child developed a maculopapular rash. Which of the following is the most likely diagnosis? A. Fifth Disease (erythema infectiosum) B. Roseola infantum (exanthema subitum) C. Varicella D. Infantile maculopapular rashes - Correct answerB. Roseola Infantum (Exanthema subitum) A 70-year old male patient complains of a bright red-colored spot in his left eye for 2 days. He denies eye pain, visual changes, or headaches. He has a new onset of cough from a recent viral upper respiratory infection. The only medicine he is on Bayer aspirin, 1 tablet a day. Which of the following is most likely? A. Corneal abrasion B. Acute bacterial conjunctivitis C. Acute uveitis D. Subconjunctival hemorrhage - Correct answerD. Subconjunctival hemorrhage. Bright red blood in a sharply defined area surrounded by normal-appearing conjunctiva indicates subconjunctival hemorrhage. A woman is being evaluated by the nurse practitioner for complaints of dyspareunia. A microscopy slide reveals a large number of atrophic squamous epithelial cells. The vagin*l pH is 4.0. There are very few leukocytes and no RBCs are seen on the wet smear. Which of the following is most likely? A. Atrophic vaginitis B. Bacterial Vaginosis C. Trichom*oniasis D. This is a normal finding. - Correct answerA. Atrophic vaginitis. Symptoms of atrophic vaginitis include painful intercourse, atrophic squamous epithelial cells, and a decrease in pH. vagin*l atrophy is caused by lack of or imbalance of estrogen. Normal pH of the vagin* is 4.0-5.0 (acidic) The most common type of skin malignancy is: A. Squamous cell cancer B. Basal skin cancer C. Melanoma D. Dysplastic nevi - Correct answerB. Basal skin cancer. It is several times more common that squamous cell skin cancer according to the NIH. Koplik's spots are associated with: A. Poxvirius infections B. Measles C. Kawasaki's disease D. Reye's syndrome - Correct answerB. Measles. S/S include fever over 101, coryza, cough, conjunctivitis, rash, and koplik's spots on buccal mucosa positive Murphy's inspriatory arrest. Ilcerative colitis presents with LLQ pain. Pain associated with pancreatitis is typically in the epigastric region. Atenolol (Tenormin) should be avoided in: - Correct answera 43-yo female with asthma. Atenolol is a beta blocker. It should be avoided in pateints with asthma or other bronchospastic conditions. These patients often require beta stimulation, not beta blockade. Although atenolol is a beta 1-selective agent, at higher doses beta 2 receptors (bronchial and vascular) are blocked. Beta blockers, specifically atenolol, are beneficial post-myocardial infarction because they have demonstrated a reduction in morbidity and mortality. All of the following interventions with pediatric patients are appropriate EXCEPT: - Correct answerPre-medicate the patient prior to all painful interventions. Pre-medication is not warranted prior to all painful interventions. For example, premedication is not warranted prior to routine immunizations but it certainly is prior to suturing. Pain management should be an integral part of patient management. Infants with veliac disease (gluten eneteropathy) are at risk for multiple complications. The most urgen complication of this disease is: - Correct answerIntususseption or volvulus. Intuseusseption and volvulus are surgical emergencies. Delay releasing the invagin*ted or "telescoped" bowel (intususseption), or releasing the twisted bowel (volvulus) may result in tissue death and gangrene, perforation, peritonitis, and/or sepsis, and fatality. There is also a high rate of intususseption and volvulus among infants with cistic fibrosis. Which activities are NOT characteristic of preschool children? - Correct answerAlways follow rules during playground games. Since preschoolers are just beginning to learn moral behaviors, they often cheat to win. While most preschoolers toilet independently, accidents occasionally occur and bed- wetting is not unusual. The use of a security item such as a blanket is common. A 38-yo pregnant patient at 18 weeks gestation, complains of feeling light-headed when standing. Which of the following is an appropriate response by the NP? - Correct answerBlood pressure normally decreases during pregnancy and can cause this symptom. Blood pressure normally decreases during pregnancy, reaching the lowest point during the second or third trimesters and rising there after. Patient education to rise slowly from sitting or lying is important. Low blood glucose may be the etiology, but an oral glucose tolerance test at this point is not indicated. A fasting blood glucose could be ordered, however, an electrocardiogram is not indicated. An 80 year old Caucasian female has heart failure. What symptom is an early indicator of failure? - Correct answerWeight gain. Early signs of heart failure include weight gain (the most sensitive indicator), S3 gallop, dyspnea on exertion, peripheral edema. Moderate signs include nocturnal cough, tachycardia. Late signs include ascities, frothy sputum, and hypotension. A 6 yo had an acute onset of fever, pharyngitis, and headache 2 days ago. Today, he presents with cervical lymphadenopathy and sandpaper textured rash everywhere except on his face. A rapid streptococcal antigen test is positive. The remainder of the assessment in unremarkable. What is the most likely diagnosis and the most appropriate action? - Correct answerScarlet fever; treat with antibiotics. This disease is due to infection with Group A Beta-hemolytic streptococcus. The rash is thought to be due to a systemic reaction to the toxin produced by the microorganism. The rash fades with pressure and ultimately desquamates. A deep, nonblanching rash on the flexor surfaces of the skin is referred to as pastia lines. A patient has been diagnosed with hypothyroidism and thyroid hormone replacement therapy is prescribed. When should the nurse practitioner check the patient's TSH? - Correct answer6 weeks. The half-life of levothyroxine, the treatment of choice for thyroid replacement, is 7 days. The earliest that meaningful changes will be observed is at 4-6 weeks. Therefore, the NP should wait a minimum of 4-6 weeks before checking the patient's TSH. A 15 yo malue has a history of cryptorchidism which was surgically repaired. Because of this information, it is essential for the nurse practitioner to teach him about: - Correct answertesticular self-examination. Cryptorchidism, even with surgical repair, is associated with increased risk for testicular cancer. The treatment of choice for chronic bacterial prostatitis (CBP) is: - Correct answera flouroquinolone twice daily for 3 weeks to 4 months. The treatment of chice is a flouroquinolone twice daily for 3 weeks to 4 months. The cure rate with Bactrim-DS is only about 30-40%. A 25 yo female has a history of frequent candidal vagin*l infections in the past year. She is in a monogamous sexual relationship and uses and IUD for contraception. Of the following, which is the most likely underlying conidition predisposing her to recurring candidal vaginitis? - Correct answerDiabetes. A common underlying cause of frequent infections is diabetes mellitus. Pregnancy increases the incidence of candidiasis, but is unlikely a factor with this patient. Which of the following is NOT a characteristic of the S3 heart sound? - Correct answerThe sound is high-pitched and occurs just prior to the S1 heart sound. The S3 heart sound is low-pitched and occurs just after the S2 heart sound. It is produced by rapid ventricular filling and is best auscultated in the mitral area. It is a common finding with right-sided heart failure, rapid growth, and the last trimester of pregnancy. Following the finding of prostate gland abnormalities on DRE, the NP orders the appropriate labs. Whem preparing to review lab reports with the patient, the nurse practitioner knows all of the following are true EXCEPT: - Correct answernormal PSA is 10ng/ml or less. Normal PSA is 4ng/ml or less. PSA levels greater the 4 and less than 10 are associated with BPH. A 10 or greater PSA level suggests prostate cancer. Positive serum acid phosphatase is associated with malignancy of the prostate gland with bone metasasis. A 66 yo female presents to your clinic. She states that yesterday evening she had chest pain for 20-30 minutes. Which finding most strongly correlates with myocardial infarction? - Correct answerElevated Troponin I levels An elevated creatinine kinase (CK) is not diagnostic of a myocardial infarction (MI). CK may be elevated from an IM injection, surgery, ot any type of extensive skeletal muscle trauma or prolonged, strenuous physical exertion. ST segment depression on EKG usually indicates an ischemic myocardium, but, not necessarily, one post-MI. Elevated ST seghments reflect mycardial damage. MB bands are specific for myocardial smooth muscle. If these are elevated, the patient MAY HAVE had a very recent MI. The most accurate marker of cardiac damage, because it is more specific and sensitive than CK MB, is a troponin measurement. What is a secondary cause of hyperlipidemia? - Correct answerhypothyroidism Hypothyroidism is a common secondary cause of hyperlipidemia. In the evaluation of a patient with hyperlipidemia, a TSH should always be checked and corrected before attempting treatment for hyperlipidemia. Other possible causes of seconday hyperlipidemia include pregnancy, excessive weight gain, excessive alcohol intake, insulin resistance or deficiency, obstructive liver disease, and uremia. Some medications can produce secondary hypothyroidism too: thiazide diuretics, some beta- blockers, oral contraceptives, and corticosteroids. A 35-yo male presents with a complaint of low pelvic pain, dysuria, hesitancy, urgency, and reduced for of stream. The nurse practitioner suspects acute bacterial prostatitis. The NP would appropriately collect all of the following specimens EXCEPT a: sterile in- and-out catheter urine specimen. - Correct answerA sterile in-and-out catheter specimen would identify only organisms in the bladder and would not differentiate between bladder, kidney, or prostate site infection. The sequence for obtaining specimens when prostat infection is suspected is: 1. voided urethral urine, 2. Voided mid-stream bladder urine, and 3. voided post-prostate massage urine. A 24 yo female taking an oral contraceptive has missed her last 2 pills. What should the nurse practitioner advise her to do to minimize her risk of pregnancy? - Correct Low frequency hearing loss is more commonly associated with Meniere's disease. MRI is helpful to rule out acoustic neurome, but not to diagnose Meniere's disease. The reason beta-adrenergic blockers should be avoided in patients with diabetes is because they may: - Correct answerMask symptoms of hypoglycemia. Beta blockers may mask the peripheral signs of hypoglycemia like jitteriness and tachycardia. However, beta blockers will not mask diaphoresis. Therefore diabetics on betablockers should be taught to look for this specific symptom as a possible indication of hypoglycemia. An adult female patient is seeking information about her ideal weight. She is 5 feet 7 inches tall. Using the "height-weight formula" what is her ideal body weight? - Correct answer135 lbs The height-weight formula is a quick method of determining ideal weight. Females allow 100 lbs for the first 5 feet of height plus 5 lbs for each additional inch. Males allow 106 lbs for the first 5 feet plus 6 lbs for each additional inch. This method can only be used as an estimate because it does not account for body composition or age. The nurs practitioner is caring for a 19 year old female college student with iron deficiency anemia secondary to heavy menstrual bleeding. An appropriate INITIAL treatment for this patient is: - Correct answerOral ferrous sulfate. With iron deficiency anemia, iron stores of the body must be replenished as well as the underlying cause corrected. A daily iron supplement is used initially. The most common form is ferrous sulfate. Intramuscular iron dextran is usually not needed, but may be required in the presence of a malabsorption disorder, inflammatory bowel disease, intolerance to oral iron, or blood loss too great to be compensated by oral iron. A male patient with chronic atrial fibrillation takes a generic brand of Coumadin (warfarin). He should report all of these to his health care provider EXCEPT: - Correct answerone missed dose of warfarin. This patient takes warfarin for prevention of emboli secondary to chronic atrial fibrillation. According to the NHLBI and ACCP, warfarin is the standard of care when anticoagulation is required for this condition. Warfarin is a drug with a narrow therapeutic index. This means that fluctuation in its level (increased or decreased) can potentiall cause big changes in its therapeutic effect. Consequently, warfarin levels are checked frequently to maintain therapeutic levels. If changes such as medication substitutions occur, warfarin levels should be checked in 3-7 days. Levels should be checked minimally every 4-6 weeks once regulation has occurred. While it is important NOT to omit doses of warfarin, this is the least important of all the choices because the effect of warfarin lasts beyond 24 hours. Which of the following medication or medication class that does NOT have dizziness or vertigo as potential adverse side effects? - Correct answerMeclizine (Antivert) Dizziness is associated with a sensation of body movement when there is no body movement occurring. (The person is spinning and the room is not). Vertigo is the sensation that the person is still and the room is spinning. There is no associated muscle weakeness or visual disturbance with either of these conditions. The most common causes are related to drug ingestion, hypotension, inner or middle ear pathology, and positional vertigo. Dizziness is an adverse reaction associated with certain antibiotics (gentomycin and strptomycin) and high-dose salicylates. Vertigo is associated with certain inner ear pathology such as labyrinthitis and Meniere's disease. Meclizine is a long-acting antihistamine which is used to treat chronic vertigo. A PPD is considered positive at 5 millimeters or more for which population? - Correct answerConfirmed or suspected HIV infection, injecting drug users, close contacts of a TB case, persons with a chest X-ray suggestive of TB A PPD greater than or equal to 10 mm is considered positive for injecting drug users known to be HIV negative, occupants of long-term care facilities, age less than 4 years, groups with a high prevalance for TB, the medically underserved, and healthcare workers. A PPD is considered positive at 15mm or more in those with no known risk factors. Considering mortality statistics for the adolescent age group. education targeted toward this group should first focus on: - Correct answeralcohol abuse. Alcohol is the most commonly used psychoactive substance in the United States today. It has been used by about 90% of adolescents by 16 years of age. Motor vehicle accidents related to driving under the influence of alcohol are the leading cause of death in the 15-24 year old age group. A patient with moderate persistent asthma will probably be most effectively managed with daily: - Correct answerinhaled steroids and long acting bronchodilators. A patient with moderate persistent asthma has symptoms daily. He is best managed with daily medications of inhaled corticosteroids and long acting bronchdilators. Oral leukotriene blockers may be added to this regimen. A child has just scalded her index finger with hot water at home. The mother calls the NP within 5 minutes of the injury. All of the following are appropiate instructions for the mother regarding the care of the patient with a 2nd degree burn EXCEPT: - Correct answerApplying butter, cooking oil, or lanolin for pain relief. Flushing first or second degree burn with cool water is appropriate to prevent further thermal injury and to provide pain relief. Oil should NEVER be applied to a burn injury. Consider consultation for burns in patients who are under 10 years of age and over 50 years of age. Physician referral is recommended for all 3rd degree burns, for 2nd and 3rd degree burns involving more than 10% of the body surface area, any deep thickness burns involving more than 2% of the body surface area, and burns involving the face. What is the recommended treatment for cat scratch disease? - Correct answerusually self-limiting and presents with lymphadenopathy that goes away on it's own; severe cases may benefit from azithromycin What is the most common cause of traveler's diarrhea? - Correct answere coli What bacteria is responsible for causing cat scratch disease? - Correct answerbartonella henselae What are the 4 different components of tetralogy of fallot? - Correct answeroverriding aorta, pulmonary stenosis, right ventricular hypertrophy, and a ventricular septal defect What does the murmur auscultated in tetralogy of fallot sound like? - Correct answersystolic murmur best heard at the left upper sternal border What does the murmur auscultated in a ventricular septal defect sound like? - Correct answersystolic murmur best heard at the left lower sternal border Where is the aorta best auscultated? - Correct answerat the second ICS to to the right of the sternum Where is the pulmonary artery best auscultated? - Correct answerat the second ICS to the left of the sternum What is a scotoma? - Correct answera blind spot or aura that obstructs part of your vision Is presbyopia nearsightedness or farsightedness? - Correct answerfarsightedness meaning it can be harder to see things up close, such as reading a newspaper and may need to hold it farther away or use reading glasses Is myopia nearsightedness or farsightedness? - Correct answernearsightedness meaning it is hard to see things far away Do cataracts cause myopia or presbyopia? - Correct answermyopia; meaning nearsightedness and they make things blurry that are farther away which is why driving at night becomes difficult What is the USPSTF's recommendation on scoliosis screening for adolescents? - Correct answerinsufficient When should adolescent girls be screened for scoliosis? - Correct answeras early as 10 years old and then again at age 12 What is the recommended treatment for Kawasaki disease? - Correct answerrefer to the ED, will likely need IVIG and high dose aspirin What are the s/s of Kawasaki disease? - Correct answerfever + CREAM (conjunctivitis, rash, erythema of palms and soles, adenopathy cervical, mucous membranes) Kawasaki disease is caused by what? - Correct answeran acute vasculitis of the small and medium blood vessels What is the main difference between Kawasaki disease and scarlet fever? - Correct answerKD has conjunctivitis and scarlet fever does not The rash associated with acute rheumatic fever - Correct answererythema marginatum The rash associated with parvovirus aka slapped cheek aka fifth disease - Correct answererythema infectiosum What does erythema marginatum of rheumatic fever look like? - Correct answererythematous with a central clearing, not itchy The medical term for tennis elbow - Correct answerlateral epicondylitis What is the leading cause of death in anorexia? - Correct answersuicide Erikson stage from birth to 18 months - Correct answertrust vs mistrust Erikson stage from 18 months to 3 years - Correct answerautonomy vs shame and doubt Erikson stage from 3-5 years - Correct answerinitiative vs guilt Erikson stage from 5-13 years - Correct answerindustry vs inferiority Erikson stage from 13-21 years - Correct answeridentity vs role confusion Erikson stage from 21-39 years - Correct answerintimacy vs isolation Erikson stage from 40-65 years - Correct answergenerativity vs stagnation Erikson stage for 65 and older - Correct answerego integrity vs despair Macrobid/nitrofurantoin is contraindicated in which patients? - Correct answerthose with renal insufficiency Treatment options for complicated UTI - Correct answercipro or levo x 5-7 days Treatment options for uncomplicated UTI - Correct answerbactrim x 3 days, macrobid x 5 days, fosfamycin 1g one time dose Olecranon bursitis is pain in which region of the elbow? - Correct answerthe posterior elbow; will also have swelling and warmth Lateral epicondylitis manifests as what type of elbow pain? - Correct answerweak grip and pain with gripping, pain with rotation of the wrist and forearm The medical term for golfer's elbow - Correct answermedial epicondylitis How does the pain in lateral vs. medial epicondylitis differ? - Correct answerlateral epicondylitis is worse with supination of the hand and medial epicondylitis is worse with pronation of the hand involves tapping a finger along the problem nerve - Correct answertinel sign asking the patient to place both elbows on a table while keeping both forearms vertical and flexing both wrists - Correct answerphalen sign Which nerve runs on the side of the thumb - Correct answermedian nerve Which nerve runs on the side of the pinky finger - Correct answerulnar nerve What is ankylosing spondylitis? - Correct answeran inflammatory condition affecting the spine causing chronic back pain and stiffness that improves with exercise and enthesitis What is the treatment for ankylosing spondylitis? - Correct answerrefer to rheumatology for possible DMARDs What are the s/s of polymyalgia rheumatica? - Correct answerchronic pain in the proximal joints (shoulders and hips) bilaterally with stiffness lasting for more than 45 minutes What is the recommended treatment for polymyalgia rheumatica? - Correct answerPO steroids and refer to Rheumatology to confirm the diagnosis Where are the biggest food sources of vitamin B12? - Correct answerfish, meat, poultry, eggs, and dairy products, also fortified breakfast cereals and fortified nutritional yeasts What is the gold standard for the diagnosis of herpes? - Correct answernucleic acid amplification test Most cases of acute sinusitis are viral and how can we treat them? - Correct answertreat the symptoms with intranasal steroids and saline rinses What is the difference between chronic vs gestational hypertension? - Correct answerchronic htn is present before 20 weeks gestation or 12 weeks after delivery; gestational hypertension is after 20 weeks gestation Which is the recommended diagnostic when a patient has a breast lump? - Correct answerif less than 45, do an ultrasound. if older than 45, do a diagnostic mammogram. An overactive bladder is what type of incontinence? - Correct answerurge incontinence What is the recommended treatment for urge incontinence? - Correct answeranticholinergics like oxybutynin What is the recommended treatment for stress incontinence? - Correct answeralpha agonists like sudafed, TCAs What is the recommended treatment for BPH? - Correct answeralpha blockers like terazosin, doxazosin, tamsulosin Alcohol and marijuana worsen which type of incontinence? - Correct answerurge Which SSRI is safest in pregnancy? Which poses the most risk? - Correct answersertraline; paroxetine Infants born to women who smoke are at increased risk for what? - Correct answerlow birth weight and preterm delivery What is the best marker of an acute MI? - Correct answertroponin When do troponins peak after a MI? - Correct answer12 hours Used to establish a designated level of professional competence - Correct answerlicensure Treatment of subclinical hypothyroidism is generally not indicated unless the TSH is ___________. - Correct answergreater than 10 In the presence of an elevated serum TSH, what is the next step in management? - Correct answerrepeat TSH plus free T4 Patients with a history of UC or Crohn's should have a colonoscopy how often? - Correct answerevery 1-5 years due to the increased risk of colorectal cancer What is the first line treatment for community acquired atypical pneumonia? - Correct answermacrolides such azithromycin. If the pt cannot take a macrolide, then they can do levofloxacin or doxycycline What is metatarsus adductus? - Correct answerthe medical term for in-toeing What is true regarding older adults who are overweight? - Correct answermortality in older adults related to overweight states declines over time How do we manage rubella immunity in pregnant women? - Correct answerwe screen all pregnant women with a rubella titer. If they had a protective rubella titer in a previous pregnancy, then they do not need to be checked again. If they are found to not be immune when pregnant, then she should be vaccinated AFTER delivery How is permethrin applied to the skin for scabies? - Correct answera one time application to all skin below the neck and then left on for 8-12 hours is sufficient After exposure to HIV, when will the patient seroconvert and become positive? - Correct answer3 months Is age less than 18 years a risk factor for an ectopic pregnancy? - Correct answerno; but personal history of PID, an IUD, previous ectopic pregnancy, previous tubal surgery, pathology, or ligation all are risk factors What is Tanner stage 5 of breast development in females? - Correct answerprojection of the nipple only What are two common classes of drugs that interact with grapefruit juice? - Correct answerstatins and CCBs Ankle inversion is a common complaint from a patient with what? - Correct answera lateral ankle sprain How can we best diagnose coarctation of the aorta? - Correct answerby comparing the upper and lower extremity blood pressures What is the most important risk factor for developing breast cancer in women of average risk? - Correct answerage hey - Correct answerstopped at top of page 7 What is the recommended treatment for blepharitis? - Correct answererythromycin 1cm ribbon 5x daily for 7 days What is the recommended treatment for severe anorexia with delusions? - Correct answer2nd atypical antipsychotics What lab values need to be monitored in a patient on Tizanidine? - Correct answerLFTs and BP at baseline and 1 month after the max dose is achieved (can cause hepatotoxicity and hypotension) What class of drug is dabigatran (pradaxa)? - Correct answerdirect thrombin inhibitor The most effective treatments for acne related to increased sebum production are what? - Correct answeroral contraceptives and isotretinoin Oral contraceptive regimen in which both the estrogen and progesterone concentrations vary throughout the cycle - Correct answerquadriphasic Oral contraceptive regimen in which the estrogen and progestin doses are fixed throughout the cycle - Correct answermonophasic Oral contraceptive regimen in which in the estrogen dose remains the same for the first 21 days of the cycle, while progestin is lower in the first half and higher in the second half - Correct answerbiphasic Oral contraceptive regimen in which the estrogen remains consistent and the progestin dose varies - Correct answertriphasic Patients who are receiving beta blockers may find what treatment for allergies ineffective? - Correct answertreatments containing epinephrine For a patient on warfarin who needs to receive a tetracycline antibiotic, how will we adjust their warfarin? - Correct answertetracyclines depress prothrombin activity, same as warfarin, so we need to decrease the warfarin to avoid an increase in INR and significant bleeding What is the herbal, valerian root often used for? - Correct answerinsomnia/sleep, ADHD, anxiety, depression What is the herbal, saw palmetto, often used for? - Correct answerBPH What is the herbal, black kohosh, often used for? - Correct answersymptoms of menopause - hot flashes What is the herbal, St. John's wart, often used for? - Correct answerdepression What are the intrauterine transplacental infections that can be passed from mother to infant? - Correct answerthink TORCH; toxoplasmosis, Other (syphilis, varicella zoster, parvovirus) Rubella, Cytomegalovirus and Herpes simplex, as well as tuberculosis. Note that chlamydia is NOT transplacental, but rather transvagin*l during childbirth The antibiotic of choice for a dog bite - Correct answeraugmentin; doxy if there is a penicillin allergy Obese women should gain how much weight during pregnancy? - Correct answer11-20 pounds Overweight women should gain how much weight during pregnancy? - Correct answer15-25 pounds Normal weight people should gain how much weight during pregnancy? - Correct answer25-35 pounds Underweight people should gain how much weight during pregnancy? - Correct answer28-40 pounds Pregnant women should increase their daily caloric intake by how many calories per day? - Correct answer300 Immunizations are considered to be what level of prevention? - Correct answerprimary Children can start to ride a tricycle around age _____ - Correct answer3 When should pediatrics be screened for hyperlipidemia? - Correct answerbetween 9 and 11 years old What is the tell-tale symptom of endometriosis? - Correct answerprogressive dysmenorrhea What is treatment for first-line endometriosis? - Correct answercyclic OCPs and NSAIDs How is endometriosis diagnosed? - Correct answervia transvagin*l ultrasound What are risk factors for the development of cervical cancer? - Correct answerHIV, immunocompromised, low socioeconomic status, smoking, hispanic, DES daughters, long term contraceptive use What does the D in melanoma stand for? - Correct answergreater than 6mm in diameter or 1/4 of an inch Boutonniere deformities are indicative of what disease? - Correct answerrheumatoid arthritis What are the potassium sparing diuretics? - Correct answerspironolactone, triamterene, amiloride, eplenerone What class of diuretic is bumetanide? - Correct answera loop diuretic, same as lasix What is Chovstek's sign? - Correct answera contraction of ipsilateral facial muscles subsequent to percussion over the facial nerve, indicative of hypocalcemia What is Trousseau's sign? - Correct answera carpopedal spasm induced by ischemia secondary to the inflation of a sphygmomanometer cuff, indicative of hypocalcemia Calcium is in an inverse relationship with what electrolyte? - Correct answerphosphorous When you see the trigger word "christmas tree", what rash should you consider? - Correct answerpityriasis rosea What is the recommended treatment for pityriasis rosea? - Correct answernothing; it is a viral rash and can last up to two months before clearing on it's own Causing harm or damage - Correct answerdeleterious How do you calculate ideal body weight for men? - Correct answerstart at 106 pounds and then add 6 for every inch above 5 feet How is lichen sclerosus treated? - Correct answertopical steroids, topical calcineuronin inhibitors to suppress the immune system, topical or oral retinoids, phototherapy What is the tell-tale sign of malignant hypertension? - Correct answerretinopathy and blurred vision; characterized by a diastolic BP of greater than 130 What is the difference between a Parkinsonian tremor and an essential tremor? - Correct answera Parkinson tremor is also known as a resting tremor and is seen at rest, may look like pill-rolling. an essential tremor is bilateral, symmetric and seen when the hands are doing something What can worsen and improve an essential tremor? - Correct answerworsened by stress or fatigue, improved by alcohol What is first-line treatment for trigeminal neuralgia? - Correct answercarbamazepine (tegretol) or amtriptyline Are Parkinsonian tremors unilateral or bilateral at first? - Correct answerunilateral Where is the TMJ best palpated? - Correct answeranterior to the tragus What are the primary bacteria in dog bites? - Correct answerstaph, strep, and pasteurella What is the gold standard treatment of giant cell arteritis? - Correct answeroral steroids and potential artery biopsy by rheum Who is a candidate for Truvada for Prep? - Correct answerif you are an IV drug user, have had a sexual partner with HIV in the last 6 months, do not consistently use a condom and engage in anal or vagin*l sex, or have been diagnosed with a STD in the last 6 months Post exposure prophylaxis for someone with HIV should be completed when? - Correct answerwithin 72 hours of the exposure Who should be screened for HIV? - Correct answereveryone at least once from age 13-64 if low risk, and then more frequently if high risk Should patients undergo gonorrhea testing to confirm a cure after receiving treatment? - Correct answeronly if they had pharyngeal gonorrhea, not vagin*l or anal Tanner stage 1 female breast developmental - Correct answerprepubertal Tanner stage 2 female breast development - Correct answerthelarche; the breasts bud as a small mound and the areola increases in diameter Tanner stage 3 female breast development - Correct answerthe areola, breast, and nipple grow together in one mount Tanner stage 4 female breast development - Correct answerthe areola and nipple separate from the breast to form a secondary mound Tanner stage 5 female breast development - Correct answermature nipples project only Tanner stage 1 male genital development - Correct answerprepubertal Tanner stage 2 male genital development - Correct answertesticl*s and scrotum enlarge Tanner stage 3 male genital development - Correct answertesticl*s continue to enlarge and the penis lengthens Tanner stage 4 male genital development - Correct answerpenis lengthens, testicl*s enlarge, and the scrotum darkens Tanner stage 5 male genital development - Correct answermature Antibiotic of choice for lyme disease - Correct answerdoxycycline Antibiotic of choice for cellulitis - Correct answerkeflex or cefdinir Antibiotic of choice for chlamydia - Correct answerazithromycin Antibiotic of choice for syphilis - Correct answerpenicillin G Antibiotic of choice for bacterial vaginosis - Correct answerflagyl Antibiotic of choice for trichom*oniasis - Correct answerflagyl Antibiotic of choice for acute prostatitis - Correct answercipro (or ceftriaxone if STI related) On microscopic exam, what is most likely to be seen in the presence of a dermatophytic (fungal) infection? - Correct answerhyphae that are long, thin, and branching (such as in the case of tinea pedis, tinea cruris, and tinea corporis) The medical term for ringworm - Correct answertinea corporis What is the most common cause of viral pneumonia in peds? - Correct answerRSV Viral pneumonia is the most common type of pneumonia in children of what ages? - Correct answer6 months to 5 years What is the simplest way to screen for nutritional adequacy in older adults? - Correct answermeasure their weight How are scabies treated? - Correct answervia permethrin and good hygiene What are common side effects of NSAIDs? - Correct answerrenal impairment, acute renal failure, gastritis What is the most common cause of pernicious anemias? - Correct answervitamin B12 deficiency How should oral iron supplements be taken? - Correct answeron an empty stomach with something acidic and high in calcium such as orange juice Group A strep pharyngitis can be accompanied by what symptoms other than the classic Centor criteria? - Correct answerabdominal pain, nausea, and vomiting A stool specimen that is positive for WBCs is a sign of what? - Correct answera viral or bacterial infection The only type of incontinence that would not benefit from pelvic floor therapy is what? - Correct answeroverflow incontinence What do the skin manifestations of a penicillin allergy look like? - Correct answerhives What are symptoms that someone will experience as soon as they are started on a SSRI and when will this resolve? - Correct answerheadache, nausea, insomnia, restlessness, agitation; will resolve in 2 weeks What causes the dry cough in people taking an ACEI? - Correct answerthe buildup of bradykinin which is usually degraded by ACE ACEI induced coughs usually begin how soon after starting the medication? - Correct answerwithin 2 weeks COPD is often associated with what sequelae? - Correct answeranxiety and depression When should an antiviral be started in a patient with shingles? - Correct answerwithin 72 hours of the onset of symptoms What is the drug of choice for treating UTIs in male patients? - Correct answerbactrim for 7-10 days What is the leading cause of CKD and ESRD in the united states? - Correct answerdiabetes What is the usual recommendation about administering MMR and varicella immunizations? - Correct answerthey should be given on the same day or at least one month apart What are the classic symptoms of an atopic pregnancy? - Correct answeramenorrhea, vagin*l bleeding, abdominal pain What is sideroblastic anemia? - Correct answerwhen the bone marrow is unable to incorporate iron into hemoglobin. This may be either inherited or acquired. Can a pregnant woman get the MMR vaccine while pregnant? - Correct answeryes Why are the carotid arteries auscultated for bruits? - Correct answerthis is indicative of generalized atherosclerosis What is roseola? - Correct answera viral exanthem characterized by a high fever for 3 days then abrupt stopping of the fever and a maculopapular rash that blanches. also known as exanthem subitum. What are signs indicative of a meningitis infection? - Correct answerKernig and Brudzinski's signs Cancer in the bone marrow that results in skeletal destruction - Correct answermultiple myeloma What is the average age of diagnosis of multiple myeloma? - Correct answer65 a skin disorder that causes the nose to enlarge and become red, bumpy, and bulbous - Correct answerrhinophyma What are physical exam findings common in alcohol abuse? - Correct answertremors, macrocytosis (due to vitamin B12 and folate deficiencies common in alcoholics), hypertension, rhinophyma, peripheral neuropathy, teleangectasia, hepatosplenomegaly What is the first sign of sexual maturation in a male? - Correct answeran increase in testicular size (tanner stage 2) The inability to name a familiar object - Correct answeranomia What happens to a womans vital signs when she is in her second trimester of pregnancy? - Correct answerher blood pressure decreases What is the most common cause of acute pharyngitis in children? - Correct answerrespiratory viruses A depressed patient is started on an SSRI. When should another antidepressant be tried if there is no response? - Correct answer8-12 weeks If the number of bacteria on a urinalysis are greater than ____________________, this is indicative of a UTI. - Correct answer100,000 What are signs of a ruptured TM other than the obviously ruptured TM? - Correct answermuffled hearing and bright red blood in the canal Osgood Schlatter disease produces pain where? - Correct answerin the knees Growing pains usually occur when? - Correct answerat nighttime Children with complaints of midbone pain should be worked for what MNM differential? - Correct answeracute lymphocytic leukemia NSAIDs should be used in caution with what patient population? - Correct answeranyone with hypertension because they cause sodium retention and increase blood pressure What is the most common cause of secondary dysmenorrhea? - Correct answerendometriosis; symptoms include spotting in between periods, cervical stenosis or displacement, nodular or tender uterosacral ligaments What is the most common cause of hemorrhagic stroke? - Correct answerhypertension What helps prevent infection by maintaining the acidic environment of the ear canal? - Correct answercerumen Mortality associated with Kawasaki disease is most likely related to what? - Correct answercoronary artery aneurysm What is Kawasaki disease? - Correct answeran acute febrile illness of early childhood characterized by vasculitis of the medium sized arteries, most noticeable in the coronary vessels Serial echos must be performed in a patient with Kawasaki disease in order to rule out coronary artery aneurysms. When should these occur? - Correct answerat the time of diagnosis, 1-2 weeks after onset of illness, 5-6 weeks after onset of illness What is the most common cause of croup? - Correct answerparainfluenza virus What are the two subsets of impetigo? - Correct answerbullous (fluid filled) or nonbullous (honey crusts) Which protein is an important mediator of the inflammatory response and serves as a messenger at the site of antigen (allergen) invasion? - Correct answercytokines; they are released by mast cells in response to an invading antigen and recruit leukocytes and eosinophils Paroxysmal events associated with abnormal electrical discharges of neurons in the brain - Correct answerseizures What is HELLP syndrome in pregnancy? - Correct answerstands for hemolysis, elevated liver enzymes, and low platelets; is characterized by thrombocytopenia, hemolytic anemia, and liver dysfunction HELLP syndrome in pregnancy is due to what? - Correct answerendothelial dysfunction Patients with acute coronary syndrome typically have plaque erosion caused by what? - Correct answeraccumulation of hyaluronan and neutrophils Which diagnostic study is most helpful for differentiating Alzheimer's disease from dementia with Lewy bodies? - Correct answerPET scan ADHD is a risk factor for both depression and what else? - Correct answersubstance abuse Epididymitis with orchitis is commonly related to what? - Correct answergenitourinary infections Mycoplasma genitalium is most commonly associated with which disease presentation in men? - Correct answerurethritis, especially if persistent or recurrent C. trachomatis stands for what common type of bacteria infection? - Correct answerchlamydia Chlamydia passed from a mother to a newborn can result in which problems for the newborn? - Correct answerconjunctivitis and pneumonia In a patient with patent ductus arteriosus, where is the murmur best heard? - Correct answerlocalized at the left clavicle and is both systolic and diastolic What is the ductus arteriosus? - Correct answerprovides the fetus with a pathway from the pulmonary artery to the aorta in utero in order to skip the lungs What are examples of some conditions that have multifactorial inheritance? - Correct answerdiabetes, obesity, arthritis, cancer, Alzheimers Seborrheic dermatitis is basically the same thing as what? - Correct answereczema of the scalp The flow of air coming in and out of the lungs in one inhalation and one exhalation - Correct answerpeak expiratory flow What does an incentive spirometer measure? - Correct answerpeak expiratory flow What happens in the luteal phase of the menstrual cycle? - Correct answerLH and FSH decrease, the ruptured follicle closes after an egg has been released and forms the corpus luteum, which produces progesterone. Estrogen and progesterone remain high and it is those two things that cause the lining of the uterus to thicken further in preparation for fertilization What is a sign of the initial appearance of measurable histamines in a patient with allergic rhinitis? - Correct answersneezing The bacteria responsible for syphilis is called what? - Correct answertreponema pallidum The STI, chancroid, is caused by what responsible bacteria? - Correct answerhaemophilus ducreyi What is the most common cause of septic arthritis? - Correct answerstaph aureus The STI, mycoplasma genitalium, is unique in what way? - Correct answerit lacks a cell wall and has a terminal tip-like structure which allows it to attach to and enter other cells. It then modulates the host's immune response What are the 3 secondary causes of type 2 diabetes? - Correct answerprolonged physical or emotional stress, pregnancy, drugs such as steroids and hormonal contraceptives How does hyperinsulinemia in PCOS work? - Correct answerinsulin resistance occurs in all places except for the ovary which remains sensitive to insulin. This insulin upregulates androgen production. Which drug classes are responsible for causing urinary retention? - Correct answerantihistamines, TCAs, antispasmodics, bronchodilators, antipsychotics, carbidopa levodopa A 75 year old patient is experiencing progressive confusion and memory deterioration. Which condition mimics dementia that this patient should be tested for? - Correct answerneurosyphilis Which hormones in late pregnancy can block insulin and cause gestational diabetes? - Correct answercortisol, estrogen, and placental lactogen Target shaped skin lesions as a result of a hypersensitivity reaction to certain infections and drugs - Correct answererythema multiforme An abnormal growth of squamos epithelium in the middle ear and mastoid - Correct answercholesteatoma The cremasteric reflex is assessed in infants for what purpose? - Correct answerto assess for retractile testicl*s; this occurs when the reflex is too strong and it pulls the testicl* out of the body and into the groin; this is normal and should go away by puberty Patient populations who may present atypically when having a MI include who? - Correct answerdiabetics, females, elderly A disorder that results in the destruction of platelets due to an autoimmune response - Correct answeridiopathic thrombocytopenia purpura Fifth disease is spread via how? - Correct answerrespiratory droplets What are other names of fifth disease? - Correct answerparvovirus, slapped cheek disease What is the life span of lice? - Correct answer30 days The hypersensitivity and pain crescendo experienced in conjunction with a migraine is thought to be related to the release of what? - Correct answersubstance P; a neurotransmitter which functions as the pain transmitter of the dorsal horn and is responsible for causing nausea, vomiting, stress, and anxiety Where is the location of the tricuspid valve? - Correct answerseparates the right atrium and ventricle What does a tricuspid regurgitation murmur sound like? - Correct answerbest heard at the left sternal border, on both systole and diastole. Increases with inspiration When can you hear the murmur associated with pulmonic stenosis? - Correct answermid-late systolic murmur When can you hear the murmur associated with aortic regurg? - Correct answerdiastole only When can you hear the murmur associated with mitral valve stenosis? - Correct answerdiastole only What EKG changes may be present during episodes of angina? - Correct answertransient ST elevation, T wave changes such as inversions, ST depressionals, Q waves Children and teens present with depression most commonly how? - Correct answerirritability Are the sickling of RBCS in sickle cell anemia reversible? - Correct answerinitially, yes they are reversible with oxygenation. But repeated episodes cause the cells to become irreversibly sickled. When the brain is exposed to something rewarding, it responds by increasing the release of what? - Correct answerdopamine Where are T cells produced? - Correct answerin the thymus Where are B cells produced? - Correct answerin the bone marrow What is the pathophys behind a thyrotoxic crisis (aka thyroid storm)? - Correct answerT3 and T4 are overproduced which leads to increased production of epinephrine and severe hypermetabolism What causes a reduction in the number of RBCs in pregnancy? - Correct answeriron deficiency anemia due to blood volume expansion and a decrease in hemoglobin levels so the iron stores are depleted because they are needed to be used for the expanded blood volume When should the ductus arteriosus close in an infant? - Correct answerwithin the first 1-2 days of birth The minimum arterial pressure exerted during relaxation and dilation of the ventricles - Correct answerdiastole Lyme disease is caused by what spirochete present on ticks? - Correct answerborrelia burgdorfei What are the potential sequelae of strep pharyngitis? - Correct answeracute glomerulonephritis, peritonsillar abscess, toxic shock disease, rheumatic heart disease (although rare) The medical term for pubic lice aka crabs - Correct answerphthirus pubis Petechiae noted on the cervix during a pelvic exam is a sign of what disease process? - Correct answertrichom*oniasis; also called "strawberry cervix" What is the difference between a folliculitis and a furuncle? - Correct answerfolliculitis is an inflammation of the follice whereas a furuncle is an abscess that results from folliculitis Urea is eliminated by the kidneys, but where does it come from originally? - Correct answerfrom the liver as a byproduct of protein metabolism What are the 5 primary structures of the immune system? - Correct answerlymph nodes, thymus, tonsils, spleen, bone marrow Bacteria most commonly responsible for pyelonephritis - Correct answere coli What causes back pain in pregnancy? - Correct answercurvature of the lumbosacral vertebrae What is dacrocystitis? - Correct answerinflammation of the nasolacrimal duct due to duct obstruction or infection What are potential causes of blepharitis? - Correct answerbacterial infection, dry eyes, rosacea What are s/s of croup? - Correct answerhoarseness, cough, inspiratory stridor, possible respiratory distress due to narrowing of the larynx Women who become pregnant later in life have an increased risk of what? - Correct answerbreast cancer What are things women can do to decrease their risk of breast cancer? - Correct answerregular physical activity, breastfeeding Does an increase or decrease in blood pH affect the rate and degree of sickling in sickle cell anemia? - Correct answera decrease in blood pH increases the rate What is the difference between Prinzmetals and stable (classic angina)? - Correct answerPrinzmetals may occur randomly and while at rest, whereas stable angina is usually only with physical activity Why may infection of the palatine tonsils become serious very quickly? - Correct answerthe area is composed of loose connective tissue What are cutaneous xanthomas? - Correct answerlocalized lipid deposits caused by elevated lipids or LDL Uterine prolpase is caused by stretching of the uterine ____________ - Correct answerligaments What causes bacterial vaginosis? - Correct answera decrease in lactobacillus species within the vagin* Where is acanthosis nigricans most likely to occur? - Correct answeraxillae, neck, groin What causes subjective tinnitus? - Correct answerabnormal neuronal activity HIV produces cellular immunity deficiency by depleting what? - Correct answerhelper T cells What is an acoustic neuroma? - Correct answersomething pressing, likely a tumor, on the acoustic nerve that results in dizziness, loss of balance, facial numbness, hearing loss, and tinnitus The axillary lymph nodes drain lymphatic fluid from where? - Correct answerbreast, upper abdomen, upper back, pectoral region, and upper limbs International travelers to endemic areas are at a risk for what form of hepatitis? - Correct answerhepatitis A because it can be transmitted through contaminated food or drink Adrenarche is the early presence of pubic hair in girls aged __________ - Correct answer5-8 years old A condition that presents with symptomatic limb ischemia upon exertion is termed: - Correct answeratherosclerotic peripheral arterial disease The largest salivary gland is the parotid gland, and where does it drain to? - Correct answerStensen's duct which has an opening on the buccal mucosa opposite the second molar The varus stress test assesses for a tear in which part of the lower extremities? - Correct answerlateral meniscus The valgus stress test assesses for a tear in which part of the lower extremities? - Correct answermedical meniscus To evaluate the uterus during a bimanual exam, the examiner would lubricate the index and middle fingers of one hand and insert them into the vagin*. The other hand would be placed on the abdomen where? - Correct answerabout midway betwee the umbilicus and symphysis pubis while elevating the cervix and uterus with the other hand A feeling of constantly needing to pass stool - Correct answertenesmus Severe constipation resulting from intestinal obstruction - Correct answerobstipation What does the vagin*l discharge associated with trich look like? - Correct answerprofuse, yellowish green that is malodorous What does the discharge associated with bacterial vaginosis look like? - Correct answergray or white, thin, malodorous (fishy) and not profuse The nurse practitioner instructs the patient to place on hand behind his back and touch his shoulder blade. This shoulder movement elicits what? - Correct answerinternal rotation How do we test for flexion and extension of the shoulder? - Correct answerextension is asking the patient to move their arm behind themself. Flexion is asking the patient to move their arm in front of the body Symptoms related to Shaken Baby Syndrome are due to what? - Correct answeruncontrollable cerebral edema and hypoxia What are the symptoms of Shaken Baby Syndrome? - Correct answerblindness, inability to perform previously learned milestones, flaccid muscle tone Koplik's spots are indicative of what disease? - Correct answermeasles (rubeola) The medical term for measles - Correct answerrubeola What do Koplik's spots of measles look like? - Correct answersmall white specks that resemble grains of salt on a red background Round or oval shaped lesions surrounded by erythematous mucosa found anywhere on the oral mucosa - Correct answeraphthous ulcers Excessive or prolonged menstrual flow occurring at regular intervals - Correct answermenorrhagia Intermenstrual bleeding (aka spotting in between periods) - Correct answermetrorrhagia Polymenorrhea is when there are fewer than ________ between periods - Correct answer21 days Oligomenorrhea is when there are more than _______ between periods; or only 4-9 menstrual cycles per year - Correct answer35 days Retraction of the nipple and areola are suggestive of what? - Correct answeran underlying breast mass; this is never considered normal What is the straight leg raise test? - Correct answerused to evaluate for sciatica; the NP raises the straight leg while flexing the hip and then dorsiflexes the foot. Positive if pain is elicited. What is Tinel's sign? - Correct answerpositive for carpal tunnel; the NP taps over the median nerve and is positive if n/t is felt What is Phalen's sign? - Correct answerpositive for carpal tunnel; patient holds their wrists together to form right angles for 60 seconds and is positive if n/t is felt Test used to identify de Quervain's tenosynovitis - Correct answerFinkelsteins test Test of thumb movement by instructing the patient to place his thumb in the palm and then move the wrist toward the midline in ulnar deviation - Correct answerFinkelsteins test What does the murmur associated with mitral regurg sound like? - Correct answerpansystolic, harsh murmur heard loudest at the apex with radiation toward the left axilla When auscultating the apex of the heart in a child older than 7, the bell of the stethoscope should be placed where? - Correct answerfifth intercostal space to the left of the midclavicular line; if younger than 7, it is best heard at the third or fourth intercostal space What is the main sign of Kaposi's sarcoma? - Correct answerraised or flat deep purple lesions in the mouth Where do lesions of Kaposi's sarcoma usually manifest? - Correct answerin the mouth, GI tract, or lungs A patient is experiencing a dull achy pain in the epigastric area with eating. This type of pain is consistent with what? - Correct answera gastric ulcer How is the pain of a gastric ulcer vs. a duodenal ulcer different? - Correct answerthe pain is worsened with eating with a gastric ulcer but improved with eating in a duodenal ulcer What are s/s of glaucoma? - Correct answerloss of peripheral vision, halos around lights, difficulty focusing on near objects Higher level functioning, such as using the telephone, doing laundry, taking medicine, etc. are considered to be called what? - Correct answerinstrumental activities of daily living An inability to extend the knee is a sign of what? - Correct answera partial or complete patellar tendon tear What do bronchovesicular breath sounds sound like? - Correct answerfull inspiratory phase with a shortened and quieter expiratory phase Thenar atrophy suggests what? - Correct answera median nerve disorder Does the ulnar nerve innervate the pinky or the thumb? - Correct answerpinky The pregnant woman who has a BMI of less than 18.5 should gain how many pounds in pregnancy? - Correct answer28-40 pounds The pregnant woman who has a BMI of 18.5-24.9 (normal) should gain how many pounds in pregnancy? - Correct answer25-35 pounds The pregnant woman who has a BMI of 25-29.9 (overweight) should gain how many pounds in pregnancy? - Correct answer15-25 pounds The pregnant woman with a BMI over 30 should gain how many pounds in pregnancy? - Correct answer11-20 pounds A BMI of less than _____ is considered underweight - Correct answer18.5 The term used to refer to skin that is peeling - Correct answerexfoliation What will a basal cell carcinoma on the eyelid look like? - Correct answerpapule with an ulcerated center, most often on the lower lid and medial canthus What is Rovsing's sign? - Correct answerpositive for appendicitis; palpation on the LLQ elicits pain in the RLQ What causes plantar warts? - Correct answerhuman papillomavirus; usually occur on the sole of the foot In second degree uterine prolapse, where is the cervix located? - Correct answerat the introitus In third degree uterine prolapse, where is the cervix located? - Correct answerboth the cervix and vagin* are outside the introitus The anterior cervical lymph node chain is located where? - Correct answeranterior and superficial to the sternocleidomastoid muscle How do we diagnose a migraine headache? - Correct answerif 3 of the 5 POUND symptoms are present: pulsatile or throbbing, one day duration or 4-72 hours if left untreated, unilateral, nausea or vomiting, disabiling or intense causing interruption of daily activity What causes the S4 sound? - Correct answerthe atrium forcefully contracting against a stiffened ventricle During the vagin*l exam of a pregnant woman in her first trimester, an adnexal mass with tenderness is palpated. This is consistent with what? - Correct answeran ectopic pregnancy Stridor indicates a partial or complete obstruction of the _________ and is best heard where? - Correct answerlarynx; neck When assessing an african american person, black, dusky mucous membranes are related to what - Correct answercyanosis Persistent, sharp, and knife-like chest pain is indicative of what? - Correct answerpericarditis or pleuritic chest pain Thin, pencil-like stool can be associated with what? - Correct answera lesion in the sigmoid colon An acute viral illness that presents with a burning retrosternal discomfort and a dry cough is suggestive of what? - Correct answertracheobronchitis The shiny, pink area located around the cervical os is known as what? - Correct answersquamos epithelium Tenderness of the posterior medial malleolus may be a sign of what? - Correct answertibial tendinitis When performing a rectal exam on a female, a hard nodular rectal "shelf" is palpable at the tip of the examiner's finger. What is this suggestive of? - Correct answermetastatic tissue in the rectouterine pouch Which of these instructions should a nurse include in the teaching plan for a client who had removal of a cataract in the left eye? a. "Forcefully cough and take deep breaths every two hours to keep your airway clear." b. "Perform the prescribed eye exercises each day to strengthen your eye muscles." c. "Rinse your eyes with saline each morning to prevent postoperative infection." d. "Take the prescribed stool softener to avoid increasing intraocular pressure." - Correct answerd. "Take the prescribed stool softener to avoid increasing intraocular pressure." A client vomits during a continuous nasogastric tube feeding. A nurse should stop the feeding and take which of these actions? a. Suction the nasogastric tube. b. Flush the tube with 30 mL of sterile water. c. Remove the nasogastric tube. d. Check the residual volume. - Correct answerd. Check the residual volume. Which of these actions best demonstrates cultural sensitivity by a nurse? a. The nurse talks in a slow-paced speech. b. The nurse asks clients about their beliefs and practices toward pregnancy. c. The nurse uses charts and diagrams when teaching pregnant clients. d. The nurse can speak several different languages. - Correct answerb. The nurse asks clients about their beliefs and practices toward pregnancy. Which of these manifestations should a nurse expect to observe in a 3-month-old infant who is diagnosed with dehydration? a. Hyperreflexia. b. Tachycardia. c. Bradypnea. d. Agitation. - Correct answerb. Tachycardia. When assessing a client's risk of developing nosocomial infection, a nurse plans to determine potential entry portals, which include: a. the urinary meatus. b. vomitus. c. contaminated water. d. sexual intercourse. - Correct answera. the urinary meatus. A client who is on the inpatient psychiatric unit has a history of violence. Which of these actions should a nurse take if the client is agitated? a. Encourage the client to verbalize feelings. b. Lock the client in a secluded room. c. Ask the other clients to give feedback regarding the client's behavior. d. Ignore the client's inappropriate behavior. - Correct answera. Encourage the client to verbalize feelings. Which of these measures should a nurse include when planning care for a school-aged child during a sickle cell crisis episode? a. Monitoring for signs of bleeding. b. Providing pain relief. c. Administering cool sponge baths to reduce fevers. d. Offering a high calorie diet. - Correct answerb. Providing pain relief. Which of these instructions should a nurse include in the plan of care for a 32-week gestation client who had an amniocentesis today? b. "Please bathe the client in room 10, administer a back rub, and then evaluate if the back rub eased the client's discomfort." c. "Please measure the intake and output for the client's in rooms 8. 9. and 10, and record each on the intake/output sheets by 2 P.M." d. "Please toilet the clients in rooms 11, 12, and 13 mid-morning and after lunch." - Correct answerb. "Please bathe the client in room 10, administer a back rub, and then evaluate if the back rub eased the client's discomfort." A client has the following order for regular insulin (Humulin R) on a sliding scale: Blood sugar 150-180 mg: Give 2 units regular insulin Blood sugar 181-200 mg: Give 4 units regular insulin Blood sugar 201-220 mg: Give 6 units of regular insulin Blood sugar above 220 mg: Call MD At 11 A.M., a nurse obtains a finger stick glucose of 198 mg. The only syringe is a three milliliter one. Regular insulin is available as 100 units per milliliter. How many milliliters should the nurse administer? a. 0.04 b. 0.4 c. 4 d. 40 - Correct answera. 0.04 Which of these nursing diagnosis is the priority for a client who is one-hour postoperative after extensive abdominal surgery? a. Risk for impaired physical mobility. b. Risk for deficient fluid volume. c. Risk for ineffective airway clearance. d. Risk for infection. - Correct answerc. Risk for ineffective airway clearance. A nurse should recognize that which of these occupations increases a person's risk of developing hepatitis B? a. Sanitation worker. b. Nursery school teacher. c. Hemodialysis nurse. d. Fish market sales person. - Correct answerc. Hemodialysis nurse. Which of these assessments is the priority for a client who sustained second-degree burns of the face and neck? a. Respiratory status. b. Renal function. c. Level of pain. d. Signs of infection. - Correct answera. Respiratory status. A nurse should place a child who is two hours post-tonsillectomy and adenoidectomy in which of these positions? a. Supine, flat. b. Orthopneic. c. Trendelenberg. d. Side-lying. - Correct answerd. Side-lying. Which of these instructions should a nurse include in the discharge teaching for a client who has diabetes mellitus? a. "Soak your feet in hot water once a day." b. "Cut your toenails in an oval shape weekly." c. "Avoid using any soap on your feet." d. "Apply lotion to your feet each day." - Correct answerd. "Apply lotion to your feet each day." A nurse inadvertently administers an incorrect medication to a client. Which of these actions should the nurse take first? a. Assess the client. b. Notify the physician. c. Contact the nurse manager. d. Complete an incident report. - Correct answera. Assess the client. An elderly client who is receiving a blood transfusion develops a rapid bounding pulse and an elevated blood pressure. Which of these actions should a nurse take? a. Add a 5% dextrose solution to the line. b. Raise the head of the bed. c. Stop the transfusion. d. Measure the client's temperature. - Correct answerc. Stop the transfusion. When caring for a client who has hepatitis B, a nurse should wear: a. gloves when administering oral medications to the client. b. a gown when changing the client's position. c. gloves when removing the intravenous cannula. d. a gown when emptying the client's used bath water. - Correct answerc. gloves when removing the intravenous cannula. Which of these outcome criteria is appropriate for a client who has a nursing diagnosis of ineffective airway clearance? a. Absence of wheezing throughout the lung fields. b. Clear lung sounds on auscultation. c. Pulse oximetry level of 80%. d. Frequent coughing throughout the day. - Correct answerb. Clear lung sounds on auscultation. A doctor prescribes liquid oral iron medication for a 4-year-old child. Which of these questions should a nurse ask the child's mother to determine if the medication is being administered correctly? a. "Are you using a straw to administer the medicine?" b. "Has your child been urinating more frequently?" c. "Have you increased your child's milk intake each day?" d. "Is there a change in the color of your child's skin?" - Correct answera. "Are you using a straw to administer the medicine?" Which of these assessment findings, if present in a 4-month-old infant who has severe diarrhea, should a nurse recognize as suggestive that the infant is dehydrated? a. Bulging anterior fontanel. b. Pulse rate of 120/minute. c. Decreased urine output. d. Cyanosis of the mucus membrane. - Correct answerc. Decreased urine output. Which of these instructions should be included in the teaching plan for the parents of a 10-month-old infant who is admitted to the hospital for failure to thrive? a. Advise the mother to make sure the infant drinks the entire bottle at each feeding. b. Encourage the mother to feed the infant slowly in a quiet environment. c. Teach the mother to position the infant on the abdomen following feedings. d. Instruct the mother to play actively with the infant during bottle feedings. - Correct answerb. Encourage the mother to feed the infant slowly in a quiet environment. When a newborn is 48 hours old, a nurse notes that the child is jaundiced. The nurse should recognize which of these conditions as a probable cause of the newborn's jaundice? a. Dehydration. b. Liver immaturity. c. ABO incompatibility. d. Gallbladder immaturity. - Correct answerb. Liver immaturity. Which of these items should a nurse removed from the food tray of a client who is on a sodium-restricted diet? a. Packet of a salt substitute. b. Grapefruit juice. c. Container of jelly. d. Ketchup. - Correct answerd. Ketchup. Which of these statements, if made by a client who had a total hip replacement, would indicate a correct understanding of the postoperative instructions? a. "I will stoop carefully to pick up items from the floor." b. "I will use a raised toilet seat in the bathroom." c. "I will bend forward when tying my shoes." d. "I will put my leg through the full range of motion each day." - Correct answerb. "I will use a raised toilet seat in the bathroom." Which of these measures should a nurse include when planning care for an 88-year-old client who is admitted to the hospital with pneumonia? a. Restricting visitors to the client's immediate family members. c. The client is showing signs of pressure; press on the skin and observe for a return of color. d. The client is allergic to the soap; check the extremities for discoloration. - Correct answerc. The client is showing signs of pressure; press on the skin and observe for a return of color. A newborn is placed under fluorescent light as part of the treatment for physiologic jaundice. During the duration of the newborn's treatment, a nurse should: a. cover the newborn's closed eyes with patches. b. measure the newborn's pulse and respirations every two hours. c. keep the newborn under the light at all times, even during the feedings. d. notify the physician if the newborns stools become greenish yellow. - Correct answera. cover the newborn's closed eyes with patches. Which of these symptoms should a nurse expect to assess in a client who develops hypoglycemia? a. Fruity breath odor. b. Polyuria. c. Diaphoresis. d. Flushed skin. - Correct answerc. Diaphoresis. A client is eight hours postoperative after a transurethral resection of the prostate (TURP). Which of these observations, if noted by a nurse, indicates a complication? a. Hourly urine output of 90 mL. b. Reports of bladder spasms. c. BP 92/60 mm Hg, pulse rate 118/minute. d. Pink-tinged urine output. - Correct answerc. BP 92/60 mm Hg, pulse rate 118/minute. A nurse should assess a child who has diabetes mellitus (type 1) for symptoms of hyperglycemia, which include: a. flushed skin and thirst. b. irritability and hunger. c. sweating and jitteriness. d. lethargy and tremors. - Correct answera. flushed skin and thirst. Which of these laboratory test results should a nurse monitor for a client who is receiving intravenous heparin therapy at a rate of 1,500 units per hour for the treatment of an acute pulmonary embolism? a. Partial thromboplastin time. b. Clot retraction time. c. Platelet levels. d. Bleeding time. - Correct answera. Partial thromboplastin time. Which of these techniques should a nurse use to assess for correct placement of a nasogastric tube prior to administering a feeding? a. Aspirate 10 mL contents and measure the pH. b. Slowly inject 50 mL of saline and observe for resistance. c. Inject 20 mL of water and listen for gurgling sounds. d. Observe for bubbles after submerging the end of the tube in a cup of water. - Correct answera. Aspirate 10 mL contents and measure the pH. A client has shortness of breath when lying down and usually assumes an upright or sitting position in order to breathe more comfortably. A nurse should document this observation as: a. dyspnea. b. bradypnea. c. orthopnea. d. apnea. - Correct answerc. orthopnea. Which of these instructions should a nurse give to a client when collecting a sputum specimen? a. "Take a deep breath, then cough and spit into this container." b. "Gargle with antiseptic mouthwash before you spit into this container. c. "Spit whatever sputum you have in your mouth into this container." d. "Drink some fluids to loosen your secretions and the spit into this container." - Correct answera. "Take a deep breath, then cough and spit into this container." A client who is receiving radiation therapy has a nursing diagnosis of imbalanced nutrition: less than body requirements related to diminished taste perception and nausea. Which of these additional nursing diagnoses should a nurse consider for the client? a. Risk for aspiration. b. Ineffective protection. c. Risk for deficient fluid volume. d. Altered tissue perfusion. - Correct answerc. Risk for deficient fluid volume. Which of these menus, if chosen by a parent of a child who has celiac disease, would indicate to a nurse that the parent understands the teaching about a gluten-free diet? a. Broiled steak, baked potato, and spinach. b. Pork chop, egg noodles, and green peas. c. Fried chicken, white roll, and mixed vegetables. d. Baked macaroni with cheddar cheese and corn. - Correct answera. Broiled steak, baked potato, and spinach. Which of these statements, if made by a nurse, is non-therapeutic because it disregards a client's feelings and concerns? a. "You appear anxious and tense." b. "Everything will be okay." c. "I notice you're biting your nails." d. "I'm not sure I understand what you're saying." - Correct answerb. "Everything will be okay." A client tells a nurse, "I am so scared about the interview tomorrow. I just know I will say the wrong thing and not get the job." Which of these responses, if made by the nurse, will create a communication barrier? a. "Would you like to practice the interview?" b. "Have you thought about some possible questions that may be asked in the interview?" c. "Tell me more about your concerns." d. "You need to relax, and everything will be fine." - Correct answerd. "You need to relax, and everything will be fine." A young healthy adult, who has been exercising in hot weather, has fatigue, loss of appetite, and lightheadedness. Which of these assessments should a nurse make? a. Determine the client's preferred diet. b. Measure the client's body temperature. c. Auscultate the lungs. d. Ascertain the client's typical sleep pattern. - Correct answerb. Measure the client's body temperature. Which of these nursing measures is the priority for a child who has hemophilia and who sustains a leg injury? a. Ensuring adequate hydration for the child. b. Soaking the child's injured leg in warm water. c. Administering the missing factor VIII to the child. d. Transfusing one unit of whole blood to the child. - Correct answerc. Administering the missing factor VIII to the child. Which of these outcomes should a nurse focus on for a client who had a bronchoscopy two hours ago? a. Preventing hemorrhage. b. Preventing pneumonia. c. Preventing aspiration. d. Preventing dehydration. - Correct answerc. Preventing aspiration. A client who had a coronary artery bypass graft four days ago suddenly develops sinus tachycardia and reports shortness of breath and dizziness. Which of these interpretations and actions should a nurse take? a. This is an expected occurrence following bypass surgery; continue to monitor the client. b. This indicates normalization of the blood pressure; hold all anti-hypertensive medications. c. This may be an early sign of heart failure; notify the physician. Which of these assignments, if delegated to unlicensed assistive personnel (UAP) by a nurse, is appropriate? a. The UAP is assigned to measure a client's intake and output. b. The UAP is assigned to assess a client's lung sounds. c. The UAP is assigned to teach a client about diet restrictions. d. The UAP is assigned to change a client's postoperative wound dressing. - Correct answera. The UAP is assigned to measure a client's intake and output. A client who has a history of asthma develops an acute asthma attack. Which of these questions should a nurse ask when assessing the etiology of this attack? a. "Have you eaten any new foods recently?" b. "How many hours did you sleep last night?" c. "Are you exercising every day?" d. "Have you reduced your fluid intake recently?" - Correct answera. "Have you eaten any new foods recently?" Which of these foods should a nurse suggest that a client who is diagnosed with iron- deficiency anemia choose for dinner? a. Cooked dry beans, green leafy vegetables, and dried fruits. b. Raw cabbage, tomato juice, and cantaloupe. c. Fresh fish, peanut butter, and oatmeal. d. Cheddar cheese, enriched bread, and yellow vegetables. - Correct answera. Cooked dry beans, green leafy vegetables, and dried fruits. A nurse in a prenatal clinic performs Leopold's maneuvers on a client who is 8-months- pregnant primarily to: a. turn the fetus in the uterus. b. ease the fetus into the true pelvis. c. assessment of the location of the placenta. d. determine the fetal presentation. - Correct answerd. determine the fetal presentation. A child is brought to the clinical for serum lead screening because of ingestion of lead- based paint. Which of these manifestations, if present in the child, would indicate early signs of lead toxicity? a. Convulsive seizures. b. Behavior changes. c. Bleeding tendencies. d. Low-grade fever. - Correct answerb. Behavior changes. Which of these recommendations should a nurse make when teaching a client who is to start taking oral prednisone (Deltasone)? a. "Take this medicine at bedtime, on an empty stomach." b. "Take this medicine with a hot beverage in the evening." c. "Take this medicine in the morning, one hour before breakfast." d. "Take this medicine in the morning with food or milk." - Correct answerd. "Take this medicine in the morning with food or milk." Which of these actions should a nurse take prior to initiating prescribed antibiotic therapy for a client who has a urinary tract infection? a. Measure the body temperature. b. Cleanse the perineum. c. Weigh the client. d. Obtain a urine culture specimen. - Correct answerd. Obtain a urine culture specimen. When caring for a client who is receiving oxygen therapy via nasal cannula, a nurse should instruct the client: a. to inhale through the mouth. b. to breathe through the nose. c. to hold the catheter when coughing. d. to take quick, shallow breaths. - Correct answerb. to breathe through the nose. Each of these clients has impaired mobility related to knee surgery. Which client should a nurse assess first? a. A 20-year-old who has a sports-related injury. b. A 37-year-old who reports limited mobility. c. A 59-year-old who has a history of hypertension. d. A 70-year-old who has bilateral cataracts. - Correct answerc. A 59-year-old who has a history of hypertension. The mother of a 2-month-old tells a nurse that the baby is consuming six ounces of plain commercial formula seven times a day, plus one ounce of cereal in the morning and at bedtime. Based on this information, the nurse should conclude that the baby's diet is: a. too high in calories. b. too high in iron content. c. deficient in calcium. d. insufficient for the baby's age and weight. - Correct answerc. A 59-year-old who has a history of hypertension. A nurse plans to assess a client's recent memory. Which of these questions should the nurse include? a. "Who is your closest friend?" b. "What was the name of the school you attended?" c. "What day were you admitted to the unit?" d. "What did you have for breakfast?" - Correct answerd. "What did you have for breakfast?" A client who has a breast tumor says to a nurse, "I am so anxious. Why did I have to get sick now?" Which of these responses, if made by the nurse, is therapeutic? a. "You will need to find someone to talk over your fears on a regular basis." b. "What do you think is making you feel so anxious now?" c. "Are you aware that there are newer, more effective treatments for breast cancer?" d. "Tell me more about your concerns." - Correct answerd. "Tell me more about your concerns." Which of these actions, if taken by a nurse who is transferring a client from the bed to the chair, is correct? a. The bed is raised to a comfortable working height for the nurse. b. The wheelchair is placed perpendicular to the bed. c. The nurse stands behind the client during the transfer. d. The nurse supports the client in an upright standing position for a few moments. - Correct answerd. The nurse supports the client in an upright standing position for a few moments. A nurse should assist a pregnant client who is in the first trimester to achieve the developmental task of this stage of pregnancy, which is: a. accepting the fact that she is pregnant. b. accepting the fact that the fetus is a separate being. c. accepting that she will soon deliver the child. d. accepting that her body image has changed. - Correct answera. accepting the fact that she is pregnant. When interacting with a client who is paranoid, a nurse should: a. use touch to place the client at ease. b. maintain a caring facial expression. c. stand close to the client. d. maintain a professional attitude towards the client. - Correct answerd. maintain a professional attitude towards the client. Which of these tasks is appropriate for a nurse to delegate to a nursing assistant in an acute care unit? a. Feeding a client who was admitted with a stroke yesterday. b. Ambulating a client who was admitted with a myocardial infarction yesterday. c. Measure the blood pressure of a client who was admitted with an asthma attack yesterday. d. Suctioning the tracheostomy that was performed on a client yesterday. - Correct answerc. Measure the blood pressure of a client who was admitted with an asthma attack yesterday. Which of these techniques should a nurse plan to use with a client who is delusional? a. Explore the delusion so the client will know it is false. b. Explain clearly why the client's belief is incorrect. Which of these assessments should a nurse make of a client who had a knee replacement this morning? a. Pain. b. Signs of infection. c. Bowel movement frequency. d. Range of motion. - Correct answera. Pain. Which of these actions should a nurse take prior to assisting an elderly client to shave his face? a. Have the client sign a consent form. b. Determine what medications the client takes. c. Soften the client's skin by applying lotion. d. Cleanse the face with a bactericidal solution. - Correct answerb. Determine what medications the client takes. Which of these factors should a nurse consider when delegating tasks to unlicensed assistive personnel (UAP)? a. The UAP's relationship with clients. b. The UAP's willingness to perform tasks. c. The UAP's previous experiences on the unit. d. The UAP's duration of employment on the unit. - Correct answerc. The UAP's previous experiences on the unit. Which of these nursing diagnoses is the priority for a young adult client who has first- degree burns of the legs and smoke inhalation from a fire in the home? a. Pain. b. Risk for infection. c. Impaired gas exchange. d. Body image disturbance. - Correct answerc. Impaired gas exchange. A child who has cystic fibrosis is receiving pancrelipase (Pancrease MT) with meals and snacks. To determine if the desired effects of the Pancrease are achieved, a nurse should consider which of these questions? a. Is the child's blood sugar level within normal limits? b. Has the child's appetite improved with the medications? c. Are the child's stools of normal consistency? d. Does the child report increased belching and flatus? - Correct answerc. Are the child's stools of normal consistency? When assessing a group of children, a nurse should recognize which child is at increased risk of developing acute glomerulonephritis? a. A 3-year-old who has multiple urinary tract anomalies. b. A 4-year-old who had a streptococcal infection a week ago. c. A 5-year-old who has recurrent enuresis at night. d. A 6-year-old who had chicken pox infection two weeks ago. - Correct answerb. A 4-year-old who had a streptococcal infection a week ago. A client says to a nurse, "I am Alexander the Great. I am a world leader and must return to my kingdom. I am not taking any medications. I do not want anyone to come near me. I need to protect myself if they do." Which of these problems should the nurse focus on first? a. Risk for violence. b. Delusions of grandeur. c. Disturbed personal identity. d. Risk for noncompliance. - Correct answera. Risk for violence. When a client who has a diagnosis of depression is taking a monoamine oxidase (MAO) inhibitor, which of these dieatry instructions should a nurse give to the client? a. "Increase your intake of foods that are high in vitamin C, such as oranges." b. "Avoid foods that contain tyramine, such as aged cheeses." c. "Increase your intake of foods high in tryptophan, such as fish." d. "Restrict foods high in sodium, such as canned soups." - Correct answerb. "Avoid foods that contain tyramine, such as aged cheeses." Which of these strategies should a nurse plan for a client who is manic and has lost 30 pounds? a. Nutritious finger foods. b. Low-protein diets. c. Limiting fluids in between meals. d. Daily weights. - Correct answera. Nutritious finger foods. A 15-year-old child who has type I diabetes mellitus receives an injection of regular insulin 5 units and isophane (NPH) insulin 15 units subcutaneously at 7:00 A.M. before eating breakfast. At 10:30 A.M., the child tells the school nurse, "I am sweating and feel weak." Which of these actions should the nurse take first? a. Measure the blood sugar. b. Determine what the child ate for breakfast. c. Give a simple carbohydrate. d. Contact the physician. - Correct answera. Measure the blood sugar. A client who has a head injury is drowsy and lethargic, and has clear nasal discharge. Which of these actions should a nurse take? a. Obtain a specimen of the drainage for culture and sensitivity. b. Test the drainage for glucose. c. Cover the nares with sterile gauze. d. Cleanse the nostrils with sterile saline solution. - Correct answerb. Test the drainage for glucose. Which of these actions, if taken by a nursing assistant, should a nurse recognize as increasing the client's risk of developing a nosocomial infection? a. Wearing non-sterile gloves while emptying the Foley drainage bag. b. Taping a paper bag to the side rail for tissue disposal. c. Placing the Foley catheter drainage bag on the bed while transferring the client. d. Using the same cuff to measure the blood pressures of all the clients on the unit. - Correct answerc. Placing the Foley catheter drainage bag on the bed while transferring the client. A nurse is preparing a client for a vagin*l examination. Which of these statements should the nurse make? a. "Go into the bathroom and empty your bladder." b. "Cleanse your perineal area with betadine solution." c. "Hold your breath while the speculum remains in place." d. "Push down as the doctor inserts the speculum." - Correct answera. "Go into the bathroom and empty your bladder." A licensed practical nurse (LPN) is assigned to care for all of these clients. Which client should the nurse assess first? a. A 25-year-old client who is terminally ill with metastatic testicular cancer. b. A 37-year-old client who has second-degree burns on both feet. c. A 49-year-old client who has an acute myocardial infarction related to cocaine ingestion. d. A 68-year-old client who is bed bound related to severe Parkinson's disease. - Correct answerc. A 49-year-old client who has an acute myocardial infarction related to cocaine ingestion. Which of these preventative measures should a nurse manager in a long-term care facility plan to institute to decrease clients' risks for falls? a. Monitoring clients frequently for evidence of activity intolerance. b. Placing all client personal items in the bedside drawers. c. Raising the side rails for all clients who have memory impairment. d. Maintaining all client beds in the highest position. - Correct answera. Monitoring clients frequently for evidence of activity intolerance. Which of these assessment findings, if present in a primigravida, indicates that the client is experiencing true labor? a. The pains are felt in the lower abdomen, back, and groin. b. The Braxton-Hicks contractions have become stronger and more frequent. c. There is an increased amount of white mucus discharge. d. There is a progressive increase in effacement and cervical dilatation. - Correct answerd. There is a progressive increase in effacement and cervical dilatation. A client is admitted for opiate detoxification for the fifth time. Which of these statements, if made by a staff member, indicates a biased view of the client? d. condition of the uterine vascular bed. - Correct answera. location of the placenta. A nurse is planning to interview a client who speaks limited English. Which of these strategies should the nurse include? a. Smile frequently during the interview interview to reduce the client's anxiety. b. Observe the client for indicators of confusion or not understanding questions. c. Maintain constant eye contact throughout the interview. d. Keep the interview short to decrease the client's fatigue. - Correct answerb. Observe the client for indicators of confusion or not understanding questions. A nurse takes the weight of a normal 2-year-old child who comes in to the pediatric clinic for a well-child visit. If the child weighted 7 lbs, 2 oz. at birth, how much should the nurse expect the child to weight at this visit? a. 14 lbs, 2 oz. b. 18 lbs, 6 oz. c. 28 lbs, 8 oz. d. 45 lbs, 10 oz. - Correct answerc. 28 lbs, 8 oz. A nurse has been discussing the nutritional needs of children with a group of parents in a clinic. Which of these statements, if made by the parent of a 2-year-old child, should the nurse follow up? a. "I give my child slices of cheese as an afternoon snack." b. "I give my child a cup of skim milk as an afternoon snack." c. "I give my child some popcorn as an afternoon snack." d. "I give my child some yogurt as an afternoon snack." - Correct answerc. "I give my child some popcorn as an afternoon snack." Which of these client care situations has the greatest potential for presenting an ethical dilemma for a nurse? a. Participating in pregnancy termination procedures. b. Counseling a client who is terminally ill with AIDS. c. Discussing contraception options with adolescents. d. Caring for a client who is from a different culture than the nurse. - Correct answera. Participating in pregnancy termination procedures. Which assessment information should a nurse obtain first when a pregnant woman and her husband arrive at the Labor and Delivery Unit? a. Whether the couple attended birthing classes. b. The frequency and intensity of labor contractions. c. The number of previous pregnancies and outcomes. d. The amount and time of the client's last food intake. - Correct answerb. The frequency and intensity of labor contractions. A client who has Parkinson's disease has been identified as being at risk for falls. Which of these actions by a nurse is most likely to reduce the client's risk of falling? a. Monitor the client's blood pressure after ambulation. b. Ensure the client wears socks when ambulating. c. Encourage frequent weight-bearing exercise. d. Assign an assistant to remain with the client when ambulating. - Correct answerd. Assign an assistant to remain with the client when ambulating. A nurse determines that the therapeutic effectiveness of magnesium sulfate (MgSO4) for client who has preeclampsia is achieved when there is increased: a. urinary output. b. blood pressure. c. respiratory rate. d. uterine movement. - Correct answera. urinary output. Which of these assessments is the initial priority of a client who is one-hour postoperative after an exploratory laparotomy? a. The appearance of the client's surgical incision. b. The client's level consciousness. c. The adequacy of the client's respiratory function. d. The client's fluid and electrolyte status. - Correct answerc. The adequacy of the client's respiratory function. Which of these client reports should a nurse recognize as suggestive of hypothyroidism? a. "My hands shake whenever I reach for anything." b. "I feel cold and tired all the time." c. "I sweat whenever I walk more than one block." d. "My head aches each evening." - Correct answerb. "I feel cold and tired all the time." A nurse is monitoring a client who is taking acetylsalicylic acid (Aspirin) 975 mg daily for adverse effects, which include: a. loss of joint mobility. b. increased serum calcium levels. c. increasing heart failure. d. occult blood in the stools. - Correct answerd. occult blood in the stools. Which of these rationales explains the purpose of nasogastric tube with suction for a client who had abdominal surgery? a. Prevention of gastric decompression. b. Removal of secretions from the stomach. c. Provision of postoperative nutrition. d. Promotion of abdominal distention. - Correct answerb. Removal of secretions from the stomach. A 75-year-old client who is newly admitted to a long-term care facility has all these nursing diagnoses. Which one is the priority? a. Risk of injury. b. Anxiety. c. Sleep pattern disturbance. d. Chronic. - Correct answera. Risk of injury. A 12-month-old child is playing with the father. Which of these behaviors indicates that the child is demonstrating object permanence? a. The child transfers a toy to the other hand when given another one. b. The child returns a block to the same spot on the table. c. The child looks for a toy that the father has hidden under the table. d. The child recognizes that a ball of clay is the same when flattened out. - Correct answerc. The child looks for a toy that the father has hidden under the table. A nurse should recognize that a client's selection of which of these foods demonstrates a correct understanding of a high-fiber diet for colon cancer prevention? a. Corn muffin. b. Bran flakes. c. Raising muffin. d. Green salad. - Correct answerb. Bran flakes. Which of these discharge instructions should a nurse include for a client who has a ruptured tympanic membrane that occurred during a fall? a. "No showers or washing of the hair for the next month." b. "Avoid yawning or holding your head down." c. "Do not allow any water to enter the ear until healing is confirmed by direct visualization." d. "Avoid swallowing and coughing until your ear has healed." - Correct answerc. "Do not allow any water to enter the ear until healing is confirmed by direct visualization." Which of these nursing measures is appropriate for a client who has recurrent renal calculi? a. Weighing the client daily before breakfast. b. Measuring the blood pressure every four hours. c. Encouraging a daily intake of three liters of fluids. d. Testing the urine for protein each shift. - Correct answerc. Encouraging a daily intake of three liters of fluids. When auscultating the lungs of a woman who is admitted for severe pregnancy-induced hypertension, a nurse notes the presence of crackles and moist respirations. These assessment findings most likely indicate which of these complications? a. A convulsion is imminent. b. Pulmonary edema has developed. What are the first generation antihistamines? - Correct answerdiphenhydramine (benadryl) and chlorpeniramine (actifed) What are s/s of the secondary stage of syphilis? - Correct answerrash on hands and feet, lymphadenopathy, fever What is the recommended treatment for chronic bacterial prostatitis? - Correct answera fluoroquinolone (cipro or levo) + bactrim What is the recommended treatment for acute prostatitis? - Correct answercipro (if not STI related) or ceftriaxone What are s/s of an intraductal breast papilloma? - Correct answerclear to bloody unilateral nipple discharge (bilateral is usually benign), and also a wart like lump palpated in the nipple area If a patient has GABHS but has an allergy to penicillins, what is the second line option? - Correct answerfirst generation cephalosporins, unless the allergy is severe, then you would consider macrolides like a -mycin PDE5 inhibitors (sildenafil, tadalafil) are contraindicated in which patient populations? - Correct answerin those who are on any type of nitrate or triptan because it could result in hypotension What class of drug is sildenafil (viagra)? - Correct answera PDE5 inhibitor which can cause hypotension so you should do a full cardiac assessment before starting a patient on this and maybe do an EKG What are the symptoms of peripheral artery disease? - Correct answerthink P meaning pain, A meaning absent or weak pulses, eschar or shiny legs, intermittent claudication What is first line treatment for PAD? - Correct answerwalking and physical activity to improve circulation. second line is an aspirin or anti-platelet What should we tell our patients with PAD NOT to do? - Correct answerdo not elevate the feet; keep them down How do we diagnose PAD? - Correct answeran ABI < 7; doppler can also be used to diagnose as well but is the second choice What are s/s of peripheral vascular disease? - Correct answerthink V meaning volume overload aka edema, may ache or be uncomfortable but is not painful, bounding pulses, ruddy discoloration If a patient is on Coumadin but then they may need to go on an antibiotic for an infection and Bactrim is the drug of choice, what should you do? - Correct answerBactrim increases INR so we would want to decrease the coumadin dose while the patient is on this If a patient is on Coumadin but then they may need to go on Rifampin, what should you do? - Correct answerRifampin decreases INR so we'd want to increase the coumadin dose If a patient on coumadin's INR is 3.1-4 ,what should you do? - Correct answerdecrease the weekly dose by 5-10% If a patient on coumadin's INR is 4.1-5.0, what should you do? - Correct answerhold one dose then decrease the weekly dose by 10% If a patient's INR is greater than 5, what should you do? - Correct answerconsult cards, likely would hold two doses then decrease the weekly dose An anorexic patient will have a BMI of what? - Correct answerless than 18 What is primary amenorrhea? - Correct answerwhen the patient has never gotten their period before (there is an absence of menarche) but they have all of their secondary sex characteristics Secondary amenorrhea is a lack of menses after _________ of not having a period; but you have had one before - Correct answer3 months What do we need to do first when a patient comes in with secondary amenorrhea? - Correct answerrule out pregnancy Anorexia can put you at risk for what? - Correct answerosteoporosis, amenorrhea, cardiac damage Amenorrhea is considered a risk factor for what? - Correct answerosteoporosis What is the best indication of an anorexic patient doing better? They tell you they are eating more, they have weight gain, or they get their period back? - Correct answerthey get their period back If a pregnant woman has syphilis, what should we do? - Correct answertreat it with penicillin while she is pregnant because there is risk for spontaneous abortion What is the causative organism of mastitis? - Correct answerstaph What is the first line treatment for mastitis? - Correct answerkeflex (a cephalosporin). If that doesn't work, then try another antibiotic. If that STILL doesn't work, then refer for an ultrasound What is one of the biggest risk factors for cervical cancer? - Correct answernumerous sexual partners Preeclampsia most commonly shows up at what time in pregnancy? - Correct answeraround week 20 How do we treat UTIs in a pregnant woman? - Correct answercan either do penicillin, cephalosporin, or a macrolide (can NOT do doxy or a fluoroquinolone) When do fibrocystic breasts tend to flare up and cause tenderness and lumps? - Correct answerabout 10 days before menses What can be a sequelae of an ectopic pregnancy? - Correct answerpelvic inflammatory disease and infertility due to scarring of the fallopian tube (salpingitis) What is the Coomb's test and when is this completed? - Correct answertest given 8 weeks in to pregnancy to determine a woman's RH compatibility If a patient's Coomb's test is positive, what do we do? - Correct answernothing, she is RH positive so we do not need to treat If a patient's Coomb's test is negative, what do we do? - Correct answergive rhogam at 28 weeks and 72 hours after birth What is the only form of non-hormonal contraception other than barrier methods such as condoms? - Correct answerIUD The vagin*l ring for contraception must be taken out how often? - Correct answerevery 3 weeks If a patient is on birth control and is currently having breakthrough bleeding, then what do we do? - Correct answerincrease the progesterone How do we treat temporal arteritis? - Correct answersteroids If a patient is on birth control and they miss one pill, what should they do? - Correct answerdouble up What is myasthenia gravis? - Correct answeran autoimmune disorder that causes muscle weakness of the voluntary muscles Genu valgum is caused by a problem with what? - Correct answerthe medial collateral ligament; think valgum as in gum makes your knees stick together Genu varum is caused by a problem with what? - Correct answerthe lateral collateral ligament What are shin splints? - Correct answera medial tibial syndrome caused by overuse of the muscles How do we treat shin splints? - Correct answerrest and avoidance of exercise until the pain resolves What is the tell-tale difference between a herniated disc and sciatica? - Correct answersciatica feels better when you stand, a herniated disc feels better when you sit down When will spinal or lumbar stenosis feel better? - Correct answerwhen the patient sits down What is Korsakoff syndrome? - Correct answera deficiency in thiamine and folate caused by alcoholism What changes to the vagin* occur with aging? - Correct answerdryness, loss of rugae, less pubic hair What is a navicular fracture? - Correct answeralso known as a scaphoid fracture; this impacts the snuff box and causes tenderness there. This will have to be casted to stabilize the fracture Heberden and bouchard's nodes are indicative of what disease process? What about swan neck deformities? - Correct answerosteoarthritis; rheumatoid arthritis What is Morton's neuroma? - Correct answera mass between the 3rd and 4th toes that feels like a pebble; most common in women who wear high heels or tight shoes How is Morton's neuroma diagnosed? - Correct answervia a positive Muddler's or squeeze test causing pain What is the hook test? - Correct answerused to assess for a bicep tendon issue How do we treat any rotator cuff injuries? - Correct answerwear a sling to stabilize the arm and refer to ortho How long do migraine headaches last? - Correct answer4-72 hours How can we abort cluster headaches? - Correct answercalcium or 100% oxygen application Which type of headaches come with lacrimation and sinus symptoms? - Correct answercluster headaches What does the apprehension test assess for? - Correct answerused to assess for stability of the shoulder or knee; assesses for stability How does the parathyroid gland regulate calcium? - Correct answerwhen parathyroid hormone is released, it pulls calcium out of the blood and into the circulation in response to hypocalcemia. it also increases absorption of calcium from the intestine What is the first line treatment for trigeminal neuralgia? - Correct answercarbamazepine (tegretol) or amitriptyline. also want to do gabapentin or something for nerve pain What is the recommended treatment for Bell's palsy? - Correct answersteroids and anti-virals How is the pain of trigeminal neuralgia vs temporal arteritis different? - Correct answerin trigeminal neuralgia, the pain is like a shock hitting the side of your face whereas temporal arteritis is more like a headache What are the preventative treatment options for migraines? - Correct answerbeta blockers, amitriptyline, venlafaxine, topamax, valproate What is Addison's disease? - Correct answeradrenal insufficiency; the patient will be low in cortisol and sodium but their potassium will be high What are s/s of Addison's disease? - Correct answerbronze or discolored skin, salt cravings a rare complication of pregnancy characterized by the abnormal growth of trophoblasts, the cells that normally develop into the placenta - Correct answermolar pregnancy What bacteria is the leading cause of urethritis? - Correct answerneisseria gonorrhea What is the most common cause of hypercortisolism? - Correct answerexcessive ACTH production How do we treat developmental hip dysplasia in a child under 6 months? - Correct answerPavlik harness; if older than 6 months may need surgery It is recommended that all children younger than __________ with sickle cell anemia take daily prophylactic antibiotics. What antibiotic is this? - Correct answer5 years old; penicillin When can cervical cancer screening stop if no abnormal screening history is present? - Correct answerage 65 When should breast cancer screening begin and end with a biannual mammogram? - Correct answerage 50-74 When should colorectal cancer screening stop? - Correct answerage 85 Who qualifies for a low dose CT scan for lung cancer screening purposes? - Correct answerage 50-80 years anyone who currently smokes or has stopped smoking within the last 5 years What does an ASC-US (atypical squamos cells of undetermined significance) pap smear result necessitate? - Correct answerDo HPV testing with this. If positive, repeat in one year if age 21-24 or send for colposcopy if older than 25. If negative, repeat pap in 3 years. Which pap smear result is considered to be HPV positive? - Correct answerLSIL If a patient has LSIL with a negative HPV test, what would you do? - Correct answerrepeat in 1 year If a patient has LSIL with no HPV test or a positive HPV test, what should you do? - Correct answerrefer for colposcopy If a patient's pap results show NILM, what should you do? - Correct answerrepeat in 3 years If a patient's pap results show ASC-H or HSIL what should you do? - Correct answerrefer for colposcopy The valgus stress test assesses what? - Correct answerthe medial collateral ligament B. Start at a lower dose than what is commonly prescribed for adults, and increase the dose slowly. C. Perform a creatinine clearance test for baseline data on all person over age 70 before starting on new medications. D. Give medication with a full glass of water to promote absorption and limit harm to the kidneys. - Correct answerB. Start at a low dose than what is commonly prescribed for adults, and increase the dose slowly. A 72 year old female presents with a medical history unremarkable except for the occasional headache. She enjoyed emotional health throughout marriage and childrearing, although her family history is significant for strokes and nervous breakdowns. Since her husband died a year ago, however, she has been waking every morning at 3am, is reluctant to go out although she has family member living nearby, and feels a continual urge to sleep. She cannot discuss her late husband without crying. Differential diagnoses would include: A. Stroke, depression, and hyperthyroidism B. Depression, Unresolved grief, and hypothyroidism C. Migraine, anxiety disorder, and hypertension D. Dementia, delirium, and pna - Correct answerB. Depression, Unresolved grief, and hypothyroidism The parent of a 13-year old diagnosed with seizure disorder calls to report that the child is exhibiting symptoms of a cold, but has no fever. The nurse practitioner should advise the parent that development of fever may: A. make the seizure medication less effective B. Lower the seizure threshold C. Increase the seizure threshold D. Have no effect on the seizure threshold - Correct answerB. Lower the seizure threshold After a 3-week camping trip, an 11-year old is seen for a target lesion with central clearing, located in the inguinal area. The patient has had a severe headache, malaise, fatigue, and generalized musculoskeletal pain for several days. Pharmacologic management of this condition includes: A. trimethoprim-sulamethoxazole (Bactrim) B. Azithromycin (Zithromax) C. Metronidazole (Flagyl) D. Doxycycline (Doryx) - Correct answerD. Doxycycline An 88 year old male presents with concerns about memory loss. He feels good, takes an aspirin daily, and has no chronic diseases. He lives alone, drives his own car, and manages his financial affairs. To evaluate his memory, which of the following tests should the nurse practitioner choose? A. Folstein Mini-Mental State examination B. Geriatric Depression Scale C. Minnesota Multiphasic Personality Inventory Myers-Briggs Test - Correct answerA. Folstein Mini-mental state examination During development of a treatment plan for an 84 year old patient with hypothyroidism, the nurse practitioner has to keep in mind the worsening of: A. renal insufficiency B. Osteoporosis C. Dementia D. Anemia - Correct answerB. Osteoporosis A 16 year old female in the first month of taking Ortho-Novum 7/7/7 complains of midcycle spotting. She hasn't missed any doses and uses no other medication. Which of the following is appropriate? A. Modifying use B. Double dosing for 2 days C. Changing to Ortho-Novum 1/35 D. Providing reassurance - Correct answerD. Providing reassurance The management of choice for polymyalgia rheumatica includes: A. whirlpool B. Prednisone C. Anelgesics D. Celebrex - Correct answerB. Prednisone A 50 year old male presents with a chief complaint of malaise. Further questioning reveals that his primary concern is delayed ejacul*tion. He is currently taking the following medications: atenolol (Tenormin), 50 mg daily; paroxetine (Paxil), 20 mg daily; loratadine (Claritin-D), 1 tablet daily; and hydrochlorthoriazide (HCTZ), 25 mg daily. The most likely cause of the patient's concern would be: A. loratadine B. hydrochlorthoriazide (HCTZ) C. lisinopril (Zestril) D. paroxetine (Paxil) - Correct answerD. Paroxetine (Paxil) A 3 year old patient presents at an inner-city clinic with fever, cough, malaise, and loss of appetite. The patient lives with several relatives, including a grandmother who also has a cough. Which of the following diagnostic tests would be most appropriate for the patient? A. throat culture B. sputum culture C. Mantoux (PPD) skin test D. Cold agglutinin test - Correct answerB. Sputum culture A 25-year old presents with the chief complaint of decreased mobility and pain of the right shoulder exacerbated with movement. The patient reports that he participated in extensive house painting 24 hours prior to the onset of pain. He denies any trauma. Passive ROM is intact. No redness or ecchymosis is present. What is the next step that should be taken in order to make a diagnosis? A. Palpate structures around the shoulder B. Obtain an MRI to evaluate the shoulder C. Order an Xray of the shoulder D. Request an EMG - Correct answerA. Palpate the structures around the shoulder A 17 year old femlae has never had her menses. She is at Tanner stage III of sexual development. Her physical exam in completely normal, and her weight is appropriate for her age and height. What is the most likely diagnosis? A. Ovarian dysgenesis B. Dysmenorrhea C. Primary amenorrhea D. Secondary amenorrhea - Correct answerC. Primary amenorrhea An 18-month old child presents with a bulging, immobile tympanic membrane; T- 103F(39.4C). Assessment also reveals a grade II/VI systolic murmur at the left sternal border. After initiation of treatment for otitis media, the most appropriate intervention is to: A. Obtain an EKG B. Obtain an Echocardiogram C. Reevaluate the patient in 10 days D. Refer the patient to a cardiologist - Correct answerC. Reevaluate the patient in 10 days Which of the following is NOT an indication of preeclampsia? A. visual disturbances B. Glucosuria C. Edema of the face and hands D. Headaches - Correct answerB. Glucosuria Which of the following pharmacotherapeutics would be most important to administer to a patient who has a corneal abrasion? A. Olopatadine (Patanol) B. Cromolyn opthalmic (Crolom) C. Timolol (Timoptic) D. Gentamycin Opthalmic (Genoptic) - Correct answerD. Gentamycin Opthalmic (Genoptic) An 88 year old patient presents with right-sided weakness after being unable to rise unassisted following a fall to the bathroom floor. History includes aphasia and noncompliance with hypertension medication regimen. What is the most likely diagnosis? A. Seizure B. Right-sided Stroke C. Left-sided stroke C. Semisupine position and offer more ounces of formula less frequently D. Lateral position and start small amounts of solid food with formula - Correct answerSmaller, more frequent feedings while holding infant. Which of the following best describes psoriatic lesions in an elderly patient? A. Shiny, purple, smooth lesions B. Localized erythematous vesicles C. Erythematous plagues with central clearing D. Red, sharply defined plaques with silvery scales - Correct answerD. Red, sharply defined plaques with silvery scales A 68-yo female presents with a new onset of left-sided throbbing headache. She has noticed some spots in her visual fields that come and go. She is being treated with an NSAID for arthritis. Sedimentation rate is elevated, but all other lab values are WNL. The headache is most likely due to: A. TIA B. NSAID induced headache C. Temporal arteritis D. Glaucoma - Correct answerC. Temporal arteritis A patient with Type 1 diabetes mellitus who is on NPH and regular insulin split-dosing presents with complaints of early morning rise in fingerstick blood glucose. A review of an at-home glucose test reveals increased morning levels. After an increase in the evening insulin dose, the problem worsens. This is most likely an example of: A. Insulin resistance B. Cortisol deficiency C. the Somogyi effect D. pawn phenomenon - Correct answerC. the Somogyi effect An 18 yo college student presents for an athletic physical. When asked about current medications, she mentions that she takes "some hearb" she bought at a health food store for migrains and menstrual cramps. Which of the following herbal remedies has been commonly used for these conditions? A. Ephedra B. Echinacea C. Feverfew D. Ginko Biloba - Correct answerC. Feverfew Which of the following GI changes in associated with normal aging? A. Decreased production of gastric acid B. Decreased incidence of gallstones C. Increased salivation D. Increased esophageal emptying - Correct answerA. Decreased production of gastric acid. A 32 year old patient reports a 6-month history of intermittant symmetrical swelling of the wrists and daily morning stiffness lasting 1 hour or more in and around other joints. What is the most likely diagnosis? A. Rheumatoid arthritis B. Osteoarthritis C. Gouty arthritis D. Reiter's syndrome - Correct answerA. Rheumatoid Arthritis An 85 year old patient related that on the way to an annual physical exam, the patient suffered a sudden loss of vision in the right eye characterized by a "bunch of lights" and a feeling that " a curtain came down" The most likely diagnosis is: A. Vitreal hemorrhage B. Optic nerve lesion C. Central retinal nerve occlusion D. Retinal Detachment - Correct answerD. Retinal detachment Assessment findings that would support a diagnosis of mitral regurgitation include: A. atrial fibrillation with summation gallop B. Loud, high-pitched pansystolic murmur C. Diastolic murmur with split S1 D. Early diastolic murmur with an S3 wave. - Correct answerB. Loud, high-pitched pansystolic murmur During an employment physical exam of a 21-year old female, bruising around the areola on the breasts is noted. An appropriate health history for these findings should include all of the following EXCEPT: A. Current social relatonships B. History of present or past traumas C. Mental health status D. Socioeconomic status. - Correct answerD. Socioeconomic status. According to the American Diabetes Association, the newest standard for determining the presence of diabetes mellitus based on fasting plasma sugar level is a vlaue equal to or greater than: A. 100 mg/dl B. 126 mg/dl C. 140 mg/dl D. 180 mg/dl - Correct answerB. 126 mg/dl A nurse practitioner orders pulmonary rehabilitation for a 75 yo patient with COPD. Expected outcomes of this program include all the following EXCEPT: A. enhanced quality of life B. Increased lung capacity C. Decreased in-patient hospitalizations D. Improved exercise capacity - Correct answerB. Increased lung capacity A 2month old infant is presented for examination and immunizations. History includes an uncomplicated full-term delivery and hepatitis B virus immunization shortly after birth. Examination is unremarkable except for a diffusely erythematous (not beet red) macular rash in the diaper area., sparing the inguinal folds. No satellite lesions are noted . The infant's rash is most likely caused by. A. Candida albicans infections. B. Eczema C. Seborrheic dermatitis D. Contact dermatitis - Correct answerD. Contact dermatitis A patient with a diagnosis of diverticulosis presents with localized left lower quadrant discomfort, a palpable mass, mild leukocytosis, and T=100F (37.8C). The patient does not apear toxic and can tolerate fluids. An appropriate plan should include: A. barium enema and nothing by mouth B. Clear liquids and antibiotics C. low-fiber diet and referral to gastroenterologist D. hospital admission for IV antibiotics - Correct answerB. Clear liquids and antibiotics. Which of the following criteria differentiates a TIA from a stroke? A. focal symptoms B. Amnesia of incident C. Slowness of onset D. Absence of residual symptoms - Correct answerD. absence of residual symptoms A patient with macular degeneration has difficulty seeing objects: A. from a distance B. In the center of the visual field C. At reading distance D. In the peripheral fields - Correct answerB. In the center of the visual fields. Trigeminal neuralgia manifests itself primarily with: A. electric-shock-like unilateral facial pain B. Unilateral facial asymmetry C. Burning bilateral facial pain D. jaw pain extending to the neck - Correct answerA. Electric-shock-like unilateral facial pain A 65-yo male presents to a clinic complaining of increasing fatigue, dyspnea on exertion, and ankle edema during the day. He has a history of mild hypertension, for which he saw his physician years ago. The physician advised the patient to decrease his salt intake. On physical exam, the patient is tachycardic, positive for jugular venous distension, and positive for S3, with a systolic murmur. Chest xray reveals cardiomegaly with vascular redistribution. The nurse practitioner's diagnosis should be:

Documents

Summaries

Exercises

Exam

Lecture notes

Thesis

Study notes

Schemes

Document Store

View all

questions

Latest questions

Biology and Chemistry

Psychology and Sociology

Management

Physics

University

United States of America (USA)

United Kingdom

APEA Predictor Exam 1014 Questions and Correct Answers | Exams Nursing | Docsity (43)

Sell documents

Seller's Handbook

About us

Career

Contact us

Partners

How does Docsity work

Koofers

Español

Italiano

English

Srpski

Polski

Русский

Português

Français

Deutsch

United Kingdom

United States of America

India

Terms of Use

Cookie Policy

Cookie setup

Privacy Policy

Sitemap Resources

Sitemap Latest Documents

Sitemap Languages and Countries

Copyright © 2024 Ladybird Srl - Via Leonardo da Vinci 16, 10126, Torino, Italy - VAT 10816460017 - All rights reserved

APEA Predictor Exam 1014 Questions and Correct Answers | Exams Nursing | Docsity (2024)
Top Articles
Latest Posts
Article information

Author: Virgilio Hermann JD

Last Updated:

Views: 5601

Rating: 4 / 5 (41 voted)

Reviews: 88% of readers found this page helpful

Author information

Name: Virgilio Hermann JD

Birthday: 1997-12-21

Address: 6946 Schoen Cove, Sipesshire, MO 55944

Phone: +3763365785260

Job: Accounting Engineer

Hobby: Web surfing, Rafting, Dowsing, Stand-up comedy, Ghost hunting, Swimming, Amateur radio

Introduction: My name is Virgilio Hermann JD, I am a fine, gifted, beautiful, encouraging, kind, talented, zealous person who loves writing and wants to share my knowledge and understanding with you.